Skin and Soft Tissue Flashcards

1
Q

With regard to ultraviolet (UV) radiation, which of the following statements is correct?

A. Most of the UV radiation that reaches the earth is type B (UVB, wavelength of 290 to 320 nm).

B. Type A UV (UVA) radiation is responsible for most of the sun damage to human skin.

C. UVA is within the photoabsorption spectrum of DNA, whereas UVB is not.

D. The melanin content of the skin is the single best intrinsic factor for protecting the skin from the harmful effects of UV radiation.

E. UV radiation acts as a tumor promoter but not a tumor initiator.

A

D. The melanin content of the skin is the single best intrinsic factor for protecting the skin from the harmful effects of UV radiation.

UV radiation comprises the middle of the electromagnetic spectrum and is divided into UVA (320 to 380 nm), UVB (290 to 320 nm), and type C UV (UVC) (240 to 290 nm), whereas visible light has a wavelength of 400 to 700 nm.

UVC is virtually eliminated by stratospheric ozone and oxygen. Only 5% of solar UV emission is UVB, but it is the most carcinogenic part of the spectrum and is responsible for sunburn. Since UVB is partially eliminated by stratospheric ozone, a 1% decrease in stratospheric ozone increases UVB flux at the earth’s surface by about 3%.

More than 95% of the sun’s UV radiation that reaches the earth’s surface is UVA. Sunbeds for indoor recreational tanning emit predominantly UVA.

UV light acts both by inducing direct DNA damage and by other mechanisms, such as alteration of cellular immunity and DNA repair mechanisms.

Although UVB and UVC radiation is within the photoabsorption spectrum of DNA, UVA radiation contributes to the development of skin cancers mainly via non-DNA targets. It penetrates more deeply and affects dermal fibroblasts, which results in photoaging. Recent data suggest that UVA may also directly affect DNA; experimental studies have shown the UVA-induced development of the characteristic carcinogenic photoproduct (cyclobutane pyrimidine dimer) seen classically with DNA damage secondary to UVB.

Melanin is the most important factor in protecting the skin from the harmful effects of UV light.

Tightly woven clothing, sunscreen use, and avoidance of sun exposure also offer protection against the harmful effects of UV radiation. UV radiation can act as both a tumor initiator and a tumor promoter.

How well did you know this?
1
Not at all
2
3
4
5
Perfectly
2
Q

The most common histologic type of melanoma is:

A. Superficial spreading

B. Nodular

C. Lentigo maligna

D. Acral lentiginous

E. Desmoplastic

A

A. Superficial spreading

Superficial spreading melanoma is the most common type; it accounts for 70% of melanomas and is characterized by some degree of radial growth.

Nodular melanoma, the next most common type, accounts for about 15% of melanomas and is characterized by vertical growth with a minimal to absent radial growth phase.

Lentigo maligna melanoma, about 10% of melanomas, is characterized by an extensive radial growth phase, most commonly occurring on sun-exposed body areas in older patients, and is generally diagnosed at a thinner stage.

Acral lentiginous melanoma is the most common type of melanoma in nonwhite individuals and is usually darkly pigmented. The prognosis depends on the thickness of the lesion, not the histologic subtype per se.

How well did you know this?
1
Not at all
2
3
4
5
Perfectly
3
Q

Which statement correctly describes increased risk for the development of malignant melanoma?

A. Melanoma accounts for 50% of all skin cancer diagnoses but accounts for 10% of deaths due to skin cancer.

B. Populations that reside at further distances from the equator have a higher incidence of melanoma.

C. An adult patient with greater than 50 clinically normal appearing nevi is at an increased risk for melanoma.

D. The p16/CDKN2A tumor suppressor gene, located on chromosome 9, is implicated in 40% of cases of familial melanoma.

E. A genetic component has been implicated in the pathogenesis of melanoma, with nearly 25% of all patients diagnosed with melanoma reporting a positive family history.

A

D. The p16/CDKN2A tumor suppressor gene, located on chromosome 9, is implicated in 40% of cases of familial melanoma.

Cutaneous melanoma accounts for only 4% of skin cancer diagnoses, yet accounts for nearly 75% of skin cancer–related deaths. Multiple environmental and genetic risk factors play a role in the development of cutaneous melanoma.

Environmental factors associated with increased exposure to UVA and UVB radiations such as geographic location (closer to the equator), occupation, history of tanning bed use, and sensitivity to UV light are particularly important risk factors.

Approximately 10%–15% of individuals with a prior history of melanoma develop a second primary melanoma, placing these patients at an increased lifelong risk of melanoma.

A genetic link appears to exist, with 10%–15% of melanoma patients reporting a family history of the disease. Factors that increase the risk for melanoma include the presence of dysplastic nevus syndrome [familial atypical multiple mole melanoma syndrome or atypical mole syndrome], a clinical syndrome distinguished by the presence of numerous large dysplastic nevi usually over the trunk; xeroderma pigmentosum (characterized by mutations in genes responsible for the fidelity of DNA repair); familial retinoblastoma; and a family history of melanoma.

Dysplastic nevus syndrome can be identified in adult patients with more than 100 clinically normal-appearing nevi or in children with more than 50 clinically normal-appearing nevi. Any patient with atypical or dysplastic nevi is at an increased risk for melanoma.

Individuals at a high risk include those with two or more first-degree relatives with melanoma, two relatives of any degree if one exhibits signs of dysplastic nevus syndrome, and those with three relatives of any degree with melanoma.

As with other familial cancers, familial melanoma is characterized by an earlier age at onset and multiple tumors. Germline mutations in the CDKN2A gene, which encodes the proteins p16/INK4A and p14ARF, are the most common cause of inherited risk for melanoma with high penetrance and may be identified in up to 40% of all melanoma families with three or more affected individuals. The penetrance of this gene is variable and varies significantly with geography, with melanoma penetrance of 0.13 (0.58) in Europe, 0.5 (0.76) in the United States, and 0.32 (0.91) in Australia by the age of 50 (80). Although commercial testing is available, testing may be premature at this time because of the risk for other cancers in CDKN2A families, and factors that modify penetrance are not yet well described. In addition, the lack of correlation between CDKN2A gene carriers and the phenotypic dysplastic nevus syndrome may falsely reassure family members. Mutations of the CDK4 gene have been identified in a few melanoma kindreds. The most frequently identified gene that predisposes to melanoma is the MC1R gene associated with red hair and freckles. Variations in this gene are associated with an elevated risk for melanoma, even in patients without red hair, but such variations impart a weak susceptibility to melanoma (low penetrance) in white populations. The deleterious effect of MC1R variations is amplified by high sun exposure. Mutations in PTC, the human homolog of the Drosophila patched gene, have been identified in most patients with basal cell nevus (Gorlin) syndrome.

Mutations have also been identified in a few sporadically occurring basal cell carcinomas.

How well did you know this?
1
Not at all
2
3
4
5
Perfectly
4
Q

A 72-year-old female with a history of type 2 diabetes and obesity presents with 24h of erythema and painful swelling of her right lower extremity. On examination, her temperature is 100.5°F, heart rate is 110 beats/minute (bpm), and blood pressure is 110/65 mmHg. Severe cellulitis is noted along the dorsal aspect of her right foot to her mid-calf. She has exquisite pain beyond the margins of the erythema that is out of proportion to her examination findings. No fluctuance, bullae, or crepitus is present. Lab values reveal white blood cell count of 22,000 cells/mm3, hemoglobin 11 g/dL, C-reactive protein (CRP) 170 mg/dL, glucose 350 mg/dL, serum sodium 133 mmol/L, and serum creatinine 1.5 mg/dL. Which of the following is correct regarding her diagnosis?

A. They are most commonly classified as monomicrobial infections.

B. The Laboratory Risk Indicator for Necrotizing Fasciitis (LRINEC) score is used to diagnose necrotizing soft tissue infections (NSTIs).

C. Type I NSTIs are polymicrobial.

D. Confirmatory imaging studies [plain radiographs or computed tomography (CT)] should be obtained prior to surgical treatment.

E. Time between symptoms and antibiotic therapy is the most important factor contributing to morbidity and mortality.

A

C. Type I NSTIs are polymicrobial.

NSTIs are an uncommon diagnosis but a dangerous surgical emergency due to the significance of time and progression. The rapid progression of the disease and the systemic inflammatory response impart high morbidity and mortality if treatment is delayed.

Approximately 80% of NSTIs are polymicrobial and classified as type I infections. Various species of gram-negative rods, gram-positive cocci, and anaerobes are typically isolated in type I NSTIs, with Bacteroides fragilis and Escherichia coli being the most common aerobic and anaerobic pathogens, respectively. The most common isolates from monomicrobial infections include Streptococcus pyogenes and Clostridium perfringens.

Type II infections are caused by group A β-hemolytic Streptococcus sp. that may occur as a monomicrobial infection or in the presence of staphylococcal species.

Type III NSTIs are rare, being caused by gram-negative marine organisms, most commonly Vibrio vulnificus.

Definitive therapy for NSTI is aggressive surgical debridement of all devitalized tissue, keeping in mind that tissue necrosis extends beyond the borders of skin involvement. The most important factor contributing to morbidity and mortality is the delay in therapy related to the onset time of symptoms and the time period of operative therapy.

Although antibiotics are an essential part of treating NSTIs, surgical therapy is necessary for patients’ survival. The diagnosis of NSTI is primarily determined by a patient’s history and physical examination.

Signs and symptoms of early NSTI are similar to those of a cellulitis or abscess and may make the initial diagnosis difficult. Physical examination findings that should raise one’s suspicion of NSTI include pain out of proportion to examination findings, bullae, crepitus, pain extending beyond lines of skin erythema, and manifestations of sepsis.

Key components of a patient’s history when considering NSTI include but are not limited to disruption of normal skin barriers (lacerations, ulcers, and surgical incisions), patient risk factors [diabetes mellitus (DM), chronic obstructive pulmonary disease (COPD), and congestive heart failure (CHF)], and environmental factors (exposure to salt/fresh water and animal/human bites).

Although imaging studies such as CT and plain radiographs may show gas in soft tissues, the absence of gas does not rule out NSTI. Furthermore, obtaining radiographic studies may further delay treatment and thus should not be performed as a confirmatory test for NSTI.

If NSTI is highly suspected based on clinical diagnosis, broad-spectrum antibiotics and surgical debridement are warranted.

The LRINEC score may help increase a clinician’s suspicion of NSTI but should not be used to diagnose or rule out NSTIs. The LRINEC score components include white blood cell count, hemoglobin, CRP, serum glucose, serum creatinine, and serum sodium.

How well did you know this?
1
Not at all
2
3
4
5
Perfectly
5
Q

Which of the following is true regarding benign cystic lesions of the skin?

A. An epidermal inclusion cyst lacks a fully mature epidermis with a granular cell layer.

B. The wall of a trichilemmal cyst, usually located on the scalp, is characterized by an epidermal lining that includes a granular cell layer.

C. The most common location of a ganglion cyst is on the dorsal aspect of the wrist.

D. Malignant degeneration may occur in a dermoid cyst.

E. A pilonidal cyst results from infection in a congenital coccygeal sinus

A

C. The most common location of a ganglion cyst is on the dorsal aspect of the wrist.

A number of cystic lesions occur in the skin. Complete excision of each of the lesions listed is curative, whereas incomplete excision may lead to recurrence. When an infection is present, primary incision plus drainage with secondary excision is preferred.

The diagnosis can often be determined from the history and location of the cyst. Epidermal inclusion cysts , the most common type of cutaneous cyst, have a completely mature epidermis with a granular layer. The creamy material in the center of these cysts is keratin from desquamated cells.

The wall of a trichilemmal cyst, the second most common type and often found on the scalp, does not have a granular layer.

Ganglions are composed of connective tissue from the synovial membrane of a joint or tendon sheath and contain a thick jellylike mucinous material similar in composition to synovial fluid. Ganglions commonly occur over the tendons of the wrist, hands, and feet and may be congenital, related to trauma, or a result of arthritic conditions. They are more common in females. Sixty percent of ganglions occur on the dorsal aspect of the wrist and arise in the region of the scapholunate ligament. Asymptomatic ganglions may be treated expectantly. If treatment is needed, the initial approach can be aspiration with a large-bore needle, with or without steroid injection. Failure of this approach necessitates surgical excision, which should include removal of the pedicle of the ganglion from its origin at the involved joint or tendon sheath.

Dermoid cysts are found along the body fusion planes and usually occur over the midline abdominal and sacral regions, over the occiput, and on the nose. Malignant degeneration has not been reported.

Although in the past it was thought that pilonidal cysts result from the penetration of a congenital coccygeal sinus by an ingrown hair, which sets the stage for infec tion and cyst formation, most now believe that pilonidal cysts are acquired. The cysts result from embedded hairs in the intergluteal cleft but may occur at other locations in the body and are more common in hirsute persons. Males have a two to four times greater risk than females.

How well did you know this?
1
Not at all
2
3
4
5
Perfectly
6
Q

A 32-year-old man has multiple soft tissue masses over his trunk and extremities. He is noted to have axillary freckling and café au lait spots. Which of these statements most accurately describes his condition?

A. It is not associated with an increased risk for the development of central nervous system (CNS) tumors and lymphoma.

B. A malignant peripheral nerve sheath tumor (PNST) will develop in 50% of affected individuals.

C. Malignant PNSTs in these patients are more often multiple and occur at a younger age than do their sporadically occurring counterparts.

D. The gene responsible for this disorder is inherited in an autosomal recessive fashion.

E. It is not associated with an increased risk for the development of soft tissue sarcomas.

A

C. Malignant PNSTs in these patients are more often multiple and occur at a younger age than do their sporadically occurring counterparts.

Neurofibromatosis (NF) is a multisystem genetic disorder with characteristic cutaneous, neurologic, and bony manifestations.

Neurofibromatosis type 1 (NF1, von Recklinghausen disease) is an autosomal dominant disorder estimated to affect 1 in 3000 individuals. The NF1 gene, located on chromosome 17q11.2, encodes a protein, neurofibromin, which is important for neuroectodermal differentiation and cardiac development.

NF1 patients may have café au lait spots (six or more spots > 5 mm in children younger than 10 years or >15 mm in adults); neurofibromas (two or more); axillary or inguinal freckling; Lisch nodules (iris hamartomas, two or more); optic nerve gliomas; sphenoid dysplasia or long-bone abnormalities; cutaneous, subcutaneous, and visceral plexiform neurofibromas; and a first-degree relative with NF1. The presence of two or more of these eight characteristics confirms the clinical diagnosis of NF1.

The most common tumor is a neurofibroma (a benign PNST), and benign schwannomas and neurilemomas may also be present. Although about half of malignant PNSTs develop in patients with NF1, affected individuals have a 3%–15% lifetime risk for the development of malignant tumors, including CNS tumors, Wilms tumor, soft tissue sarcomas, and lymphomas, as well as malignant PNSTs. These tumors often occur in association with major peripheral nerve trunks.

Malignant tumors appear as enlarging soft tissue masses, variably associated with pain and other neurologic symptoms. NF1-associated malignant PNSTs may be multiple and tend to occur at a younger age than do their sporadic counterparts. Positron emission tomography (PET) may help differentiate benign neurofibromas and schwannomas from malignant tumors.

How well did you know this?
1
Not at all
2
3
4
5
Perfectly
7
Q

What condition is associated with the development of soft tissue sarcoma?

A. Familial retinoblastoma

B. Juvenile polyposis syndrome

C. von Hippel-Lindau syndrome

D. Asbestos exposure

E. Multiple endocrine neoplasia (MEN) type 1 syndrome

A

A. Familial retinoblastoma

Inherited syndromes, including retinoblastoma (also associated with osteosarcoma), Li-Fraumeni syndrome (also related to leukemia, brain, breast, and adrenocortical cancers), NF1, Gardner syndrome (familial adenomatous polyposis), Werner syndrome, and tuberous sclerosis, all confer an increased risk for soft tissue sarcoma.

Ionizing radiation is a risk factor for soft tissue sarcomas, and such tumors tend to behave in an aggressive fashion. Exposure to vinyl chloride, arsenic, and thorium dioxide are associated with the development of hepatic angiosarcomas. Chronic lymphedema also predisposes to soft tissue sarcoma in the affected extremity, predominantly angiosarcoma. Stewart-Treves syndrome is an angiosarcoma that occurs due to chronic lymphedema after mastectomy with axillary lymph node dissection.

Other acquired diseases that increase the risk for developing soft tissue sarcomas include the human immunodeficiency virus (HIV), which increases the risk of developing Kaposi’s sarcoma. Juvenile polyposis syndrome is associated with polyps of the gastrointestinal (GI) tract as well as hereditary hemorrhagic telangiectasia. Von Hippel-Lindau syndrome is associated with renal cell carcinoma, as well as pheochromocytomas and hemangioblastomas (benign CNS tumors). Asbestos is associated with mesothelioma.

MEN type 1 is associated with hyperparathyroidism, pancreatic and duodenal endocrine tumors, and pituitary adenomas, but it is not associated with an increased risk for sarcomas.

How well did you know this?
1
Not at all
2
3
4
5
Perfectly
8
Q

A 42-year-old woman has a mass in the posterior aspect of the upper part of her arm that was first noted 3 months earlier. It is not painful, and she has no associated symptoms. Magnetic resonance imaging (MRI) demonstrates a 5-cm neoplasm arising from the triceps. The best next step in the management of this patient is:

A. PET-CT

B. Fine-needle aspiration (FNA) biopsy

C. Percutaneous core needle biopsy

D. Incisional biopsy

E. Excisional biopsy

A

C. Percutaneous core needle biopsy

How well did you know this?
1
Not at all
2
3
4
5
Perfectly
9
Q

A 42-year-old woman has a mass in the posterior aspect of the upper part of her arm that was first noted 3 months earlier. It is not painful, and she has no associated symptoms. Magnetic resonance imaging (MRI) demonstrates a 5-cm neoplasm arising from the triceps.

After biopsy, a high-grade malignant fibrous histiocytoma is diagnosed. Choose the best response regarding the outcome for this type of tumor.

A. Postoperative adjuvant radiotherapy improves outcome.

B. Preoperative chemotherapy improves outcome.

C. Lymph node dissection should be performed at the time of definitive surgical treatment.

D. Grade is a more important predictor of outcome than are tumor size and location.

E. Muscle compartment resection is necessary to maximize the chance for cure.

A

A. Postoperative adjuvant radiotherapy improves outcome.

Soft tissue sarcomas develop from mesenchymal tissues such as bone, muscle, fat, and other connective tissue. In addition, the disease accounts for less than 1% of adult and 15% of pediatric malignancies. The most common sites are the extremities, which account for more than 40% of the sites of occurrence.

There are more than 50 histologic types of soft tissue sarcoma, with liposarcoma, malignant fibrous histiocytoma, and leiomyosarcoma being the most common.

For lesions smaller than 4 cm, complete excision is an appropriate diagnostic procedure. Any subcutaneous solid soft tissue tumors greater than 4 cm should be evaluated by MRI before excision. In cases where imaging cannot be obtained, core needle biopsy is an appropriate alternative. If the imaging reveals a suspicious lesion, a core needle biopsy of the mass should be performed to determine pathology before a planned excision.

When a soft tissue sarcoma of at least 4 cm is treated with an unplanned excision, the disease-free survival is lower than in those who were treated for an elective sarcoma excision. This is likely explained by a more aggressive approach for a known sarcoma with wide margins and functionsparing excisions.

Although core needle biopsies should be the initial approach for obtaining the diagnosis of a soft tissue mass, an incisional biopsy may be considered if core needle biopsy yields nondiagnostic findings. When an incisional biopsy is necessary to achieve a diagnosis or when excising smaller tumors, it is important to plan the incision properly and avoid undue contamination of tissue planes so as not to interfere with definitive surgical treatment. An incision oriented on the long axis of the limb is preferred and should be performed so that the incision and remainder of the surgical field may be completely resected at the time of definitive surgical treatment.

For the majority of soft tissue sarcomas in adults, complete surgical resection is the mainstay of treatment. Principles of surgical treatment include resection with approximately 2-cm margins of normal tissue (except vital structures) and avoidance of enucleation. Excision and amputation of muscle groups are no longer primary treatment modalities for most patients.

The usual tumor-node-metastasis (TNM) staging system is modified for sarcomas because they rarely metastasize to lymph nodes and the tumor histologic grade has a significant prognostic value. Thus sarcomas are staged using the grade-tumor-node-metastasis (GTNM) system.

Sentinel lymph node biopsy (SLNB) is not indicated for most sarcomas unless they are of a high-grade type. The histologic grade of soft tissue sarcomas is determined by cellular atypia, mitotic rate, and presence of tumor necrosis. Classification of soft tissue sarcomas is according to the tissues they mimic, not the type of tissue from which the tumor arises. When sarcomas are poorly differentiated such that no specific histogenesis can be determined, they are designated as spindle cell sarcomas or pleomorphic sarcomas.

In addition to histopathology, it is important to evaluate the presence of any chromosomal translocations as these can be accurate diagnostic markers for soft tissue sarcomas.

Postoperative adjuvant radiotherapy is beneficial in improving local control in patients with high-grade, large, and deep tumors, whereas it is probably unnecessary for patients with small (<5 cm), superficial, low-grade tumors treated by complete resection (microscopically negative margins). Preoperative radiotherapy permits a lower administered dose with a smaller treated field. However, it is associated with a higher incidence of postoperative wound complications, and treatment proceeds without knowledge of the final surgical histopathology.

Preoperative radiotherapy is preferred for patients with marginally resectable, very large, high-grade tumors to maximize the chance of a microscopically margin-negative resection and functional preservation of the limb.

With the exception of rhabdomyosarcoma and Ewing sarcoma, neoadjuvant chemotherapy is not generally beneficial. Limited data support neoadjuvant chemotherapy as justified in certain situations of carefully selected high-risk patients with large, high-grade tumors. In terms of distant recurrence and disease-specific survival, tumor size and tumor grade are equally important independent predictors of outcome.

How well did you know this?
1
Not at all
2
3
4
5
Perfectly
10
Q

A fair-skinned 68-year-old woman has a sharply demarcated 2-cm ulcerated skin lesion in an old burn scar on her forearm. What is the most appropriate treatment for this patient?

A. Topical chemotherapy

B. Topical biologic therapy

C. Surgical excision with frozen section

D. Mohs micrographic surgery

E. Radiotherapy

A

C. Surgical excision with frozen section

Cutaneous squamous cell carcinoma (SCC) appears most frequently on sun-exposed areas, with two-thirds occurring on the head or neck; typical locations include exposed portion of the ears, the lower lip at the vermillion border, the paranasal areas, the maxillary skin, and the dorsum of the hands (Figs. 13.5 and 13.6).

Risk factors for cutaneous SCC include fair skin; light-colored eyes; prior actinic keratosis; xeroderma pigmentosum; and exposure to nitrates, arsenicals, and hydrocarbons. Other risk factors associated with the disease are chronic excessive sun exposure, immunosuppression, previous trauma, and burns.

The aggressiveness of these cancers is related to the underlying cause, location, and size of the lesion. It is increased in lesions arising in areas of previous burns (Marjolin ulcer) or trauma and in lesions of the lips and perineum.

Excision of cutaneous SCC, generally with margins of 4 to 5 mm, should be accompanied by frozen section evaluation of the surgical margins. These tumors are radiosensitive. Surgery is preferred for tumors arising in scarred, traumatized, or previously irradiated skin. Large lesions may require adjuvant radiotherapy after surgical excision.

Mohs surgery may be used for lesions with clinically indistinct margins. Regional lymph node dissection for SCC is performed for clinically evident (palpable) disease.

How well did you know this?
1
Not at all
2
3
4
5
Perfectly
11
Q

Choose the correct statement regarding basal cell carcinoma.

A. It originates from the deep dermal appendages.

B. Intermittent intense exposure to UV light is a greater risk factor than exposure at a low dose per episode of a similar total dose.

C. Fifty percent of basal cell carcinoma occurs on the head and neck of an individual.

D. The risk for a second basal cell carcinoma is lower for men with index tumors on the trunk.

E. Most common type is superficial basal cell carcinoma.

A

B. Intermittent intense exposure to UV light is a greater risk factor than exposure at a low dose per episode of a similar total dose.

Basal cell carcinoma is the most common malignancy in the United States and accounts for about 80% of all skin cancers. It originates from the pluripotent basal keratinocytes of the epidermis and from hair follicles, not from the dermis.

Exposure to UV radiation is a major risk factor for basal cell carcinoma, especially recreational exposure to the sun during childhood and adolescence. Although cutaneous SCC appears to be strongly related to cumulative sun exposure, the relationship between exposure to UV radiation and risk for basal cell carcinoma, like melanoma, is more complex. The timing, pattern, and amount of exposure are significant. Other risk factors are fair skin, light-colored hair and eyes, topical arsenic exposure, and immunosuppression.

Eighty percent occur on the head or neck. The most common type of basal cell carcinoma is the nodular form, which accounts for 60% of cases, and it appears as a classic domed, pearly papule with surface telangiectasia (Fig. 13.7; E-Figures throughout this chapter are available online at www.expertconsult.inkling.com).

Other types of basal cell carcinoma include superficial (15%), which usually appears as a minimally raised pink-red patch or papule, and morpheaform (sclerosing, infiltrative), which appears as a white scarlike plaque with indistinct margins. Some basal cell carcinomas are pigmented. Basal cell carcinomas rarely metastasize, but if they are neglected or recurrent, they can be locally destructive and require extensive local treatment and reconstruction.

After an initial diagnosis of basal cell carcinoma, the risk for a second tumor is elevated tenfold. Male gender, truncal carcinomas, and older age increase a person’s risk for the development of subsequent basal cell carcinomas.

How well did you know this?
1
Not at all
2
3
4
5
Perfectly
12
Q

A 70-year-old male presents to your office with a concerning 2cm painless lump on his left hip that he noticed over the last year, which is firm and dome-shaped. An excisional biopsy is performed, demonstrating a Merkel cell carcinoma (MCC) with negative margins. Regarding MCC, which of the following is true?

A. The Merkel cell polyomavirus likely contributes to the development of most Merkel cell carcinoma.

B. A sentinel lymph node biopsy is required only or tumors >2cm in size.

C. Risk factors for Merkel cell carcinoma include having dark skin, a weakened immune system, and overexposure to UV radiation.

D. Merkel cell dense-core granules stain positively or the neuroendocrine marker neuron specific enolase.

E. Current recommendations or tumor excision are with a 0.5-cm margin or tumors <2cm in size and 2cm margins or those >2cm in size.

A

A. The Merkel cell polyomavirus likely contributes to the development of most Merkel cell carcinoma.

The mainstay of therapy for patients newly diagnosed with primary MCC remains surgical.

Current recommendations are based on the clinical size of the primary tumor and call for tumor excision with 1 cm margins or tumors that are 2 cm in size and 2 cm margins or those that are greater than 2 cm in size.

Radiotherapy has been used as monotherapy for primary tumors with reported success, but until more data become available, surgery remains the mainstay of therapy or primary MCC tumors. Furthermore, Feng et al. characterized a novel polyomavirus, the MCPyV, and suggested an association between it and the pathogenesis of MCC. This work has sparked great interest in MCC and has opened a new pathway in the study of viral tumorigenesis.

The role of chemotherapy in the treatment of MCC remains unclear.

Since nearly one-third of clinically node-negative patients harbor microscopic nodal disease, sentinel lymph node biopsy (SLN) is currently recommended for Merkel cell carcinoma at the time of wide local excision. SLN biopsy has been shown to be important in the staging and prognosis of MCC, and SLN status is included in the most recent American Joint Committee on Cancer (AJCC) staging guidelines. SLN biopsies should be examined by both hematoxyin and eosin (H&E) and immunoperoxidase staining, including CK20. If sentinel nodes are positive, completion lymph node dissection of the nodal basin followed by radiotherapy of the basin is recommended. In cases where SLN positivity is found on immunostaining but not H&E staining of the lymph node, radiotherapy without complete lymph node dissection has been suggested as sole regional therapy.

Dark skin is not a risk factor, “white skin” is. The other two are risk factors.

How well did you know this?
1
Not at all
2
3
4
5
Perfectly
13
Q

A 50-year-old fair-skinned, blue-eyed male presents to your office with a skin lesion on the pinna of his right ear concerning for a melanoma. Biopsy determines a superficial spreading melanoma with a depth of 1.1mm. What is the next step in the management of this patient?

A. Wide local excision and sentinel lymph node biopsy

B. High dose interferon alpha 2b is recommended

C. Radiation therapy

D. Excisional biopsy and a total parotidectomy

E. Complete surgical excision with a 0.5 cm negative margin

A

A. Wide local excision and sentinel lymph node biopsy

Tumor thickness is critical for establishing the prognosis in melanoma and regional metastases indicates poor prognosis. Frozen sections have no role in the diagnosis or treatment of melanoma. Once the dermis is invaded the probability of regional or distant metastases increases substantially.

Lymphoscintigraphy and sentinel lymph node biopsy became the primary method of identifying nodal drainage patterns replacing the prior suggested nodal drainage based on location.

Tragus and anterior pinna lesions were thought to metastasize to the parotid gland and anterior cervical lymph nodes, whereas posterior pinna lesions were thought to spread to the mastoid bone and occipital and posterior cervical nodes.

Complete surgical excision with 1 to 2 cm margin is the treatment of choice.

Elective neck dissection is generally not recommended for lesions less than 1mm in thickness, whereas lymphadenectomy may offer survival advantage and better local control for lesions > 1 mm in depth and positive sentinel lymph node biopsy.

Interferon alpha-2b is recommended for patients with lymph node positive disease.

EXCISION MARGIN OF MELANOMAS
T1
Depth of invasion <1mm
Margin 1cm

T2
Depth of invasion 1-2mm
Margin 1cm

T3
Depth of invasion 2.01-4mm
Margin 2cm

T4
Depth of invasion >4mm
Margin 2cm

How well did you know this?
1
Not at all
2
3
4
5
Perfectly
14
Q

An 86-year-old female presents to your office with a new 1.8cm raised pearly nodule with surface telangiectasias on her cheek. Which of the following would be the best option in her management?

A. Surgical excision with 1 cm margins

B. Cryotherapy

C. Mohs microsurgery

D. Radiation therapy

E. Topical imiquimod

A

C. Mohs microsurgery

Basal cell carcinoma could be treated either surgical or medically. Treatment options include wide local excision, Mohs microsurgery with 4 mm margins, curettage, and cryosurgery. The standard treatment for larger basal cell carcinoma is surgical excision with cure rates greater than 99% or primary lesions; how- ever for small lesions or lesions in areas such as the face, Mohs microsurgery is the preferred treatment option.

Non-surgical treatments, typically reserved for non-surgical candidates include radiotherapy, topical and photodynamic therapy.

Topic therapy includes 5- fluorouracil and imiquimod. Imiquimod, a nonspecific immune response modifier, has been approved for the treatment of superficial basal cell carcinoma smaller than 2 cm or five times per week for the duration of 6 weeks with clearance rate more than 80%.

How well did you know this?
1
Not at all
2
3
4
5
Perfectly
15
Q

A 61-year-old male presents to your office with a lesion on his lower lip. Biopsy confirms a squamous cell carcinoma, 2 cm in diameter. In regards to squamous cell carcinoma (SCC), which of the following is true?

A. Squamous cell carcinoma of the lip most often presents on the upper lip.

B. Squamous cell carcinoma is the second most common cutaneous cancer in patients who have had a kidney transplant.

C. Actinic keratosis is not a risk factor or the development of squamous cell carcinoma.

D. Lip defects involving at least one-third of the lip require regional flaps such as an Abbe flap.

E. Margins for low risk squamous cell carcinoma range from 1.0 to 2.0cm.

A

D. Lip defects involving at least one-third of the lip require regional flaps such as an Abbe flap.

In immunosuppressed transplant patients, SCC is the most common skin cancer and it tends to have a more aggressive behavior.

Lip carcinoma is the most common oral cavity cancer, with a majority o these lesions occuring on the lower lip.

A majority of patients with oral cavity carcinomas have a history of either excessive alcohol intake or tobacco use. Lip carcinoma most likely presents as an exophytic mass and diagnosis is obtained by biopsy. Risk factors for SCC include actinic keratosis, burn wound scars and chronic inflammatory wounds. Large defects that involve up to two thirds of the lip require local flaps such as Abbe or Estlander.

How well did you know this?
1
Not at all
2
3
4
5
Perfectly
16
Q

A 79-year-old male presents with 2cm scaly pink lesion on his scalp that his primary care physician has been following for the last two years.

Biopsy confirms the diagnosis of a basal cell carcinoma. Which of the following is correct in regards to basal cell carcinoma?

A. Basal cell carcinoma is the second most common form of skin cancer, after squamous cell carcinoma.

B. Superficial basal cell carcinomas are scaly, pink
to red lesions frequently confused with psoriasis
or other eczematous dermatoses.

C. Basal cell carcinomas commonly develop in
burn scars for chronic inflammatory wounds.

D. Surgical margins for low risk basal cell carcinoma range from 0.5 to 1.0cm.

E. Mohs microsurgery cannot be considered an option in the treatment of basal cell carcinomas.

A

B. Superficial basal cell carcinomas are scaly, pink
to red lesions frequently confused with psoriasis
or other eczematous dermatoses.

Basal cell carcinoma is the most common form of skin cancer followed by squamous cell carcinoma. Basal cell carcinoma often presents as pearly nodules with telangiectasias and may bleed occasionally.

A skin biopsy to establish a diagnosis is important before treatment of any skin cancer. Margins for low risk squamous cell carcinoma range from 0.5 to 1.0 cm. Margins for low risk basal cell carcinoma range from 0.3 to 0.5cm.

Additionally, Mohs microsurgery is considered an option in non-melanoma skin cancers.

Bottom line, squamous cell carcinomas most often arise in chronically damaged skin or within actinic keratosis, in burn scars, and chronic inflammatory wounds.

How well did you know this?
1
Not at all
2
3
4
5
Perfectly
17
Q

You are consulted on a 35 year-old male, who otherwise healthy, was initially admitted 26 days ago in septic shock secondary to a necrotizing soft tissue infection of the right lower extremity.

He required high-dose vasopressors or several days following emergent operative debridement and institution of broad-spectrum antibiotics.

He was recovering well in the ICU and was recently extubated. However he developed tachycardia, fever, and leukocytosis. The ICU team calls you to evaluate because they are unable to identify a source of infection. You examine his back and find a pressure sore overlying the right ischial tuberosity approximately 4 × 5 cm in size with a black eschar overlying, expressible malodorous fluid, and surrounding cellulitis.

  1. What is the stage of this pressure ulcer?

A. Stage I

B. Stage II

C. Stage III

D. Stage IV

E. Unstageable

A

E. Unstageable

Accurate staging of pressure wounds is important because it guides management. Staging is as
follows:

Stage I: Non-blanchable erythema of intact skin; impending skin ulceration.

Stage II: Partial-thickness skin loss involving epidermis and/or
dermis; ulcer is superficial and presents clinically as an abrasion, blister, or shallow crater.

Stage III: Full-thickness skin loss involving damage or necrosis of subcutaneous tissue that may extend down to, but not through, underlying fascia; ulcer presents clinically as a deep crater with or without undermining of adjacent tissue.

Stage IV: Full-thickness skin loss with extensive destruction, tissue necrosis, or damage to muscle, bone, or supporting structures.

Unstageable/Unclassified: There are also “unstageable” pressure sores that are commonly incorrectly staged. These are pressure sores with slough/eschar than need to be debrided before one can see how deep they truly are, and can then be accurately staged.

Suspected Deep issue Injury: Purple or maroon localized area of discolored intact skin or blood-filled blister due to damage of underlying soft tissue from pressure and/or shear.

How well did you know this?
1
Not at all
2
3
4
5
Perfectly
18
Q

Surgery is performed and when the black eschar is removed, purulent fluid is expressed. After extensive operative debridement to healthy tissue, you find that the wound extends to bone. Which of the following would preclude use of Negative Pressure Wound Therapy (NPWT)?

A. Chronic wounds

B. Diabetic wounds

C. Wound location

D. Meshed skin grafts

E. Wounds that require hemostasis

A

E. Wounds that require hemostasis

Negative pressure wound therapy (NPWT) is becoming a valuable resource used by surgeons to manage difficult wounds. NPWT devices consist of an adhesive semi-occlusive dressing, tubing connected to a collection canister and a vacuum source, and an interface material to distribute the vacuum (open-pore polyurethane hydrophobic foam).

Negative pressure on the sealed, airtight wound results in:
• Increased blood flow to the wound
• Removal of excess fluid that may retard cell growth and proliferation

• Micro- and macro-deformation of the wound:
• Macrode formation is the visible stretch that occurs when the sponge contracts. It serves to draw the wound edges together, provide direct and complete wound bed contact, distribute negative pressure, remove exudate and infectious materials.
• Microde formation occurs at the cellular level and leads to cell stretch. It reduces edema, promotes perfusion, and promotes granulation tissue formation by facilitating cell migration and proliferation
• Maintenance of wound homeostasis: the semi-
occlusive dressing and foam with insulation qualities minimizes evaporation, desiccation, and heat loss

The following are common indications or negative pressure wound therapy:
• Chronic, diabetic wounds or pressure ulcers
• Meshed grafts (before and after)
• Flaps
• Chronic and acute wounds
• Subacute wounds (dehisced incisions)

The following are contraindications to negative pressure wound therapy:
• Fistulae to organs/body cavities
• Necrotic tissue that has not been debrided or eschar
• Untreated osteomyelitis
• Wounds that require hemostasis
• Placing dressing on exposed blood vessels (including anastomotic sites) or organs
• Wound malignancy

Caring or the pressure sore patients involves more than addressing the wound. Wound healing in chronic wounds requires a systemic strategy, including nutritional assessment and maintenance, control of both systemic and local infection, avoidance o excessive moisture/ incontinence, pressure and muscle spasm relief, surgical debridement and wound closure.

How well did you know this?
1
Not at all
2
3
4
5
Perfectly
19
Q

Which of the following patients would be the best candidate for flap reconstruction?

A. A 48-year-old male with Stage I sacral ulcer and has no medical comorbidities

B. A 34 year-old male recovering rom brain injury with a Stage IV ischial ulcer that recently underwent treatment for osteomyelitis

C. A 74-year-old homeless male with Stage III sacral ulcer

D. A 48-year-old poorly controlled diabetic male with Stage III pressure sore overlying the right greater trochanter

E. A 56-year-old male with Stage III chronic sacral ulcer who refuses to stop smoking despite hospitalization

A

B. A 34 year-old male recovering rom brain injury with a Stage IV ischial ulcer that recently underwent treatment or osteomyelitis

Most commonly, Stage III/IV pressure sores are referred for soft tissue reconstruction. Unfortunately some of these patients are not suitable candidates for medical or social reasons. Since these surgeries are frequently fraught with complications, there are multiple patient characteristics to optimize before coverage can be considered:

• Nutritional status
• Control of medical co morbidities
• Presence of muscle spasticity (Can this be controlled with anti-spasmodics?)
• Tobacco dependence
• Social situation (assess for presence of a responsible caretaker at home—and subsequent appropriate residence at own home vs facility), appropriate specialty mattress at residence, strict regimen of frequent turning to prevent flap necrosis/failure, appropriately padded wheelchair
• If osteomyelitis is present, then the patient likely needs bony debridement and tailored IV antibiotic therapy (typically or 6 weeks) before soft tissue reconstruction can be attempted.
• Consideration of adverse drug factors like use of steroids or immunosuppressants
Control of different causes of maceration like fecal or urine incontinence
• Medical non-compliance

Once these medical/social issues are addressed, the wound can then be optimized with thorough debridement and dressing care, in preparation for flap reconstruction.

How well did you know this?
1
Not at all
2
3
4
5
Perfectly
20
Q

Multiple options for treatment of pressure sores exist. Which of the following options should be avoided?

A. Primary closure

B. Local wound care

C. Skingrafts

D. Musculocutaneous reconstruction

E. Fasciocutaneous reconstruction

A

A. Primary closure

When planning therapeutic treatment of pressure sores, the choice of closure strategy depends not only on the location, size, and depth of the ulcer, but also on the previous management strategies employed. Primary closure should be avoided.

These wounds tend to have an absence of adequate tissue and primary closure leads to tension, scarring over the original bony prominence, and dehiscence. Skin grafting has a limited success rate, as grafting tends to provide unstable coverage.

Musculocutaneous flaps provide adequate blood supply, bulky padding, and are effective in treating infected wounds.

Fasciocutaneous flaps offer an adequate blood supply, durable coverage, and low rates of functional deformity.

How well did you know this?
1
Not at all
2
3
4
5
Perfectly
21
Q

A 50-year-old Caucasian female was referred to general surgery by her primary care provider for evaluation of a mole to her left medial calf. On examination of her left medial calf, she has a 9mm diameter pigmented lesion with irregular borders and color variation.

Excisional biopsy is performed.

Pathology will most likely show which histologic subtype of melanoma?

A. Acral lentiginous

B. Desmoplastic

C. Lentigo maligna

D. Nodular

E. Superficial spreading

A

E. Superficial spreading

Seventy-five percent of all malignant melanomas are superficial spreading melanomas.

Most arise de novo but they may be associated with a pre-existing nevus. They grow radially before growing vertically. Typical locations are in sun-exposed areas, namely the back in men and legs in women.

Nodular melanomas comprise 15%to30% of melanomas and are often dome-shaped and dark. They quickly develop a vertical growth phase.

Lentigo maligna melanoma typically develops as a brown macule in sun damaged skin of older individuals and may grow radially for years before vertical growth develops.

Acral lentiginous melanoma is the rarest melanoma in caucasians but is the most common type in Asians and dark-skinned people. They are aggressive and commonly arise on palmar, plantar, subungual, and mucosal surfaces.

Desmoplastic melanoma is a rare variant that may be mistaken for a scar, fibroma other benign lesion and should be referred to an experienced dematopathologist for evaluation.

How well did you know this?
1
Not at all
2
3
4
5
Perfectly
22
Q

If her melanoma is 2.2 mm thick, the surgical skin margin for a wide local excision of the primary lesion should be at least

A. 0.5 cm

B. 1cm

C. 2cm

D. 3cm

E. 4cm

A

C. 2cm

Wide local excision is standard treatment or melanoma. Excision should be carried through skin and subcutaneous tissue down to muscle fascia.

Current recommendation is for 0.5 to 1 cm margins or melanoma in situ, 1cm margins or melanomas ≤1mm thick, 1 to 2cm for melanomas 1.01 to 2mm thick, and 2cm margins for melanomas >2mm.

A randomized multi-center trial demonstrated no benefit to 4 cm versus 2cm margins in patients with melanomas > 2 mm thick.

Other trials have corroborated these results.

How well did you know this?
1
Not at all
2
3
4
5
Perfectly
23
Q

Which of the following is a poor prognostic indicator in melanoma patients?

A. Extremity location

B. Low Breslow thickness

C. Female gender

D. Elevated LDH

E. Younger age

A

D. Elevated LDH

Evidence based prognostic indicators were carefully considered and integrated into the current AJCC staging system to provide staging that reflects disease biology.

In 2001, Balch et al. published a prospective study of 17,600 melanoma patients to determine actors that were predictive of melanoma- specific survival.

The investigators determined that patients with melanomas of the head, neck, or trunk had a significantly worse survival rate than patients with melanomas of the extremities.

Males had a poorer prognosis than females. Increasing Breslow thickness was predictive of lower survival.
Presence of ulceration was more frequent among patients with thick melanomas (63% ulceration, Breslow thickness < 4 mm) than among patients with thin melanomas (6%, Breslow thickness ≤ 1 mm).

At all thickness levels, patients with ulcerated melanomas were found to have survival curves that were similar to patients with melanomas of the next higher Breslow thickness. The investigators found a significant step-wise decrease in survival based on increasing age.

Patients with higher number of nodal metastases and patients with macrometastases (clinically palpable nodes) also had poorer survival. Seetharmu et al. (Oncology. 2011;81(5–6):403–9) found that elevated pretreatment LDH was correlated with poorer 2 year survival in Stage IV mela- noma patients. Tumor mitotic rate has also been determined to be a significant prognostic indicator.

How well did you know this?
1
Not at all
2
3
4
5
Perfectly
24
Q

Regarding sentinel lymph node biopsy in patients with melanoma, which of the following is FALSE?

A. Completion lymph node dissection is recommended to achieve regional control in melanoma patients with positive sentinel lymph nodes.

B. Sentinel lymph node biopsy is indicated for intermediate thickness melanomas (Breslow thickness, 1 to 4mm) regardless of ulceration or mitotic rate.

C. Sentinel lymph node biopsy is indicated for staging in patients with thick melanomas (4; Breslow thickness > 4 mm).

D. In patients with clinically localized melanoma, sentinel node status is the most important prognostic indicator.

E. Sentinel lymph node biopsy is mandatory in all patients with thin melanomas (< 1 mm) if there are high-risk features.

A

E. Sentinel lymph node biopsy is mandatory in all patients with thin melanomas (< 1 mm) if there are high-risk features.

Metastasis to regional lymph nodes is the most important prognostic factor in patients with early stage melanoma. The American Society of Clinical Oncology (ASCO), Society of Surgical Oncology Joint Clinical Practice Guideline, and The National Comprehensive Cancer Network (NCCN) Guidelines Version 4 (2014) recommend routine sentinel lymph node biopsy in patients with patients with intermediate thickness melanomas (Breslow thickness 1–4 mm).

Although there is less data to support use of sentinel lymph node biopsy in thick melanomas (Breslow thickness > 4 mm), sentinel node biopsy is recommended to facilitate accurate staging. ASCO guidelines state that there is insufficient evidence to support routine sentinel node biopsy in thin melanomas (Breslow thickness < 1 mm), but it may be considered in patients with high risk features. NCCN guidelines state that sentinel lymph node biopsy is not generally indicated in patients with Breslow thickness ≤ 0.75 mm.

However, consideration should be given to sentinel lymph node biopsy in patients with Breslow thickness 0.76–1 mm, especially if ulceration or mitotic rate ≥ 1 per mm is present. Completion lymph node dissection remains the standard recommendation for patients with tumor positive sentinel nodes.

Current evidence suggests that completion lymph node dissection is effective at achieving local control. The ongoing Multicenter Selective Lymphadenectomy Trial II (MSL II) is investigating whether completion lymph node dissection also improves survival.

Current AJCC staging refined N categories to account or the prognostic difference between patients with clinically palpable (macrometastasis) and clinically negative but pathologically positive sentinel nodes (micrometastasis).

How well did you know this?
1
Not at all
2
3
4
5
Perfectly
25
Q

Regarding metastatic melanoma, which of the following is FALSE?

A. Surgery or patients with Stage IV melanoma confers a survival advantage over systemic medical therapy alone.

B. The primary objective of metastatectomy or Stage IV melanoma is resection of all known disease.

C. Recurrent Stage IV disease is not a contraindication to surgery or metastatic melanoma.

D. In-transit metastases occur less than 2 cm from the primary melanoma and are located between the primary melanoma and the regional lymph node basin.

E. More than 50% of patients with Stage IV melanoma will develop brain metastases.

A

D. In-transit metastases occur less than 2 cm from the primary melanoma and are located between the primary melanoma and the regional lymph node basin.

In appropriately selected patients with Stage IV melanoma, surgery should be considered as initial therapy. Analysis of MSL -1 data demonstrated improved survival in patients with Stage IV melanoma who underwent surgery with or without systemic medical therapy compared with systemic medical therapy alone.

Multiple sites of disease and even recurrent Stage IV disease are not contraindications to resection. The most important preoperative considerations include resectability of all known disease, tumor biology such as tumor volume doubling time, and patient comorbidities.

The most common metastatic sites for melanoma are skin, lung, lymph nodes, brain, liver, and gastrointestinal tract.

In-transit metastasis is defined as intra-lymphatic tumor in skin or subcutaneous tissue more than 2 cm from the primary tumor but not beyond the nearest regional lymph node basin. The lung is a typical site of metastasis and good evidence or pulmonary metastatectomy exists.

Melanoma is the most common metastatic tumor in the small bowel and may present as pain, obstruction, bleeding, palpable mass or weight loss. More than 50% of patients with Stage IV melanoma will develop brain metastasis. Because of this, NCCN Version 4 (2014) Melanoma recommends consideration of annual brain MRI in addition to more frequent whole body PET/CT as a part of routine surveillance in patients with Stage IV disease.

How well did you know this?
1
Not at all
2
3
4
5
Perfectly
26
Q

A 47-year-old female presents with a several months history of a slowly growing painless mass in her right thigh. On physical exam, she is noted to have a 5 cm firm, non-tender mass in the right lateral mid-thigh.

Which of the following is true?

A. Routine imaging of the mass should include a PE scan.

B. Incisional biopsy should be routinely performed in order to confirm the diagnosis and identify the histological subtype.

C. The most common histologic subtypes or extremity sarcomas include liposarcoma and malignant fibrous histiocytoma.

D. Staging includes an abdominal and pelvic CT scan to evaluate for metastatic disease in this lesion.

E. Multidisciplinary evaluation is not needed for simple, early-stage sarcomas.

A

C. The most common histologic subtypes or extremity sarcomas include liposarcoma and malignant fibrous histiocytoma.

Regarding pre-operative imaging of the tumor, MRI is generally preferred for extremity sarcomas, while C tends to be preferable for abdominal and retroperitoneal sarcomas. However, there has not been demonstrated a statistically significant difference between the two modalities. High-quality cross sectional imaging is nevertheless critical for preoperative evaluation and planning. PE can give information regarding the grade, prognostication, and response to chemotherapy in select high grade, large, deep sarcomas, and can be considered for use, but does not have a role in routine evaluation of all sarcomas.

Incisional biopsy may be required, but generally speaking, or both extremity and retroperitoneal sarcomas, needle core biopsy (with or without image guidance) provides adequate tissue sampling with good diagnostic correlation to final pathology. However, when unavailable or inadequate, incisional biopsy or extremity sarcomas remains a reasonable diagnostic option.

Care should be taken if an incisional biopsy is necessary. Incisions should be oriented longitudinally and thoughtully, as re-excision of biopsy site will be necessary with definitive operation. In some selected institutions with clinical and pathologic expertise, FNA may be adequate, but should probably not be considered adequate at low-volume centers.

There are over 50 histologic subtypes of soft tissue sarcoma recognized. Overall, the most common subtypes are liposarcoma, malignant fibrous histiocytoma (MFH), and leiomyosarcoma, however the prevalence is site specific. The most common types
or extremity sarcomas are liposarcoma and MFH, while retroperitoneal sarcomas are more commonly liposarcoma and leiomyosarcoma. GIST and leiomyosarcoma are the histological types found in visceral tumors.

Overall, lung is the most common site of metastasis or extremity sarcomas. For visceral tumors, liver is more common. Chest CT is generally advocated
for extremity sarcomas to rule out metastatic disease. Due to the complexity, rarity, and histology- based therapy for soft tissue sarcomas, prior to initiating therapy, all patients should be evaluated by a multidisciplinary team with expertise in sarcoma therapy.

How well did you know this?
1
Not at all
2
3
4
5
Perfectly
27
Q

A 47-year-old female presents with a several months history of a slowly growing painless mass in her right thigh. On physical exam, she is noted to have a 5 cm firm, non-tender mass in the right lateral mid-thigh.

Regarding surgical management of this patient, which of the following is true?

A. A functional outcome is the highest priority.

B. Surgical margin of 1 to 2 cm should be sought whenever possible.

C. Regional lymph node dissection is also required.

D. Amputation should be strongly considered as the
primary therapy for most extremity sarcomas.

E. Histologic subtype has little impact on surgical
planning.

A

B. Surgical margin of 1 to 2 cm should be sought whenever possible.

Until the early 1980s, amputation remained a primary therapy or most extremity sarcomas. In 1982, Rosenberg and colleagues at the National Cancer Institute published a randomized trial of amputation vs. limb-sparing surgery plus radiation and demonstrated equivalent overall survival and acceptable local recurrence rate of 15% with limb-salvage versus 0% with amputation. Currently, limb salvage surgery can be safely performed in over 90% of patients with extremity sarcoma with excellent local recurrence rates. Proximity of critical structures (bone, nerves, blood vessels, etc.) should be considered carefully during pre-operative planning and the expected functional outcome assessed to assist in making the decision for limb salvage versus amputation.

Careful pre-operative planning is imperative or successful resection of these tumors. Failure to obtain a negative surgical margin is the most important risk factor for local recurrence. Careful scrutiny of pre-operative imaging can result in improved outcomes by allowing the surgeon to have a thorough understanding of the extent of the tumor and relationship to local structures. Functional outcomes are also improved by careful consideration of the extent of resection required. Similarly, this allows for appro- priate consultation with subspecialists including plastic or vascular surgeons pre-operatively when anticipated being necessary.

Wide margins should be the goal of therapy. A margin of 1 to 2 cm of normal tissue can help to minimize the risk of local recurrence. However, strong tissues such as fascia can severely limit the spread of most types of sarcoma into adjacent structures. Therefore, a narrow fascial margin may be acceptable where such a narrow margin of muscle or fat would likely result in an increased risk of recurrent disease. In some cases, such as retroperitoneal sarcomas, wide margins may not be feasible.
Resection of nerves and blood vessels is occasionally unavoidable, but can often be avoided by careful skeletonization of blood vessels and resection of the perineurium along with the tumor. The concept of “planned positive margin” is feasible in many cases with the use of adjuvant radiation therapy. As long as the structure of concern is not fixed to the tumor, this carefully planned positive margin provides similar outcomes to controls with negative margins. Neo- adjuvant radiation has been advocated in this setting
or “marginally resectable” tumors in order to allow a negative margin functional resection.

Histologic subtype does play an important actor in planning surgical intervention. Well-differentiated liposarcoma (formerly called atypical lipomas) have similar recurrence rate to other types of sarcoma, but metastatic disease is rare, so surgical resection can be less aggressive when necessary.

Dermatofibrosarcoma protuberans (DFSP) and myxo brosarcoma are particularly difficult due to the common finding of microscopic tentacles that extend laterally rom these lesions. DFSP has a tendency to respect fascial borders, whereas myxofibrosarcoma o en penetrates fascia and can have multifocal skip lesions, so these factors must be considered. Lymph node metastasis is rare in STS in general, but can be seen more commonly with epithelioid or clear cell variants. For this reason, sentinel lymph node biopsy can give prognostic information, so though no clear therapeutic benefit has been shown, it may be considered in this setting.

How well did you know this?
1
Not at all
2
3
4
5
Perfectly
28
Q

Regarding neoadjuvant and adjuvant therapies for soft tissue sarcoma, which of the following is true?

A. Chemotherapy is the most important therapeutic intervention for outcomes from sarcoma.

B. Metastatectomy has no role in the management of sarcomas.

C. Radiation therapy has been shown to be of benefit in the treatment of sarcomas.

D. Local recurrence is rare after appropriate therapy.

E. Gastrointestinal stromal tumors (GIST) are treated similarly to other sarcomas.

A

C. Radiation therapy has been shown to be of benefit in the treatment of sarcomas.

Radiation therapy has been repeatedly demonstrated to be of benefit in disease free survival, though not significantly different overall survival. There remains some controversy regarding the optimal timing of the radiation therapy. When comparing neoadjuvant versus adjuvant therapy, local control rate is similar.

However, preoperative radiation has been found to double wound complications in the months following surgery compared to increased rates of long term complications such as fibrosis, edema, and joint stiffness with postoperative radiation. External beam, brachytherapy, and intraoperative radiotherapy have all been used with some success.

Patients at low risk for recurrence (i.e.,small(<5cm) superficial tumors, low grade tumors, wide surgical margins) may not derive a significant benefit from radiation and may be treated with surgery alone.

Surgery is the dominant therapeutic modality for extremity, retroperitoneal, and visceral sarcomas. Complete surgical resection is the primary actor in outcomes. Only a few clinical trials have demonstrated a statistically significant improvement in outcomes with adjuvant chemotherapy. Specific histology-directed regimens have shown the most promise and have achieved excellent and sustained results in specific subtypes to include gastrointestinal stromal tumors (GIST), rhabdomyosarcoma, Ewing sarcoma, and osteosarcoma.

Patients with an isolated metastasis can be considered or metastatectomy if resection with or without chemotherapy for radiation may result in a cure. This may include removal of limited disease in a single organ or regional node dissection if nodal metastasis is isolated. Specifically in the setting of isolated lung metastasis, median survival is lengthened from 11 to 33 months compared to observation.

Patients with widely metastatic disease may be considered
for palliation using a variety of modalities including surgery, chemotherapy, radiation, embolization, and ablation procedures.

Local recurrence rate varies with the site of the initial tumor and the adequacy of surgical resection, but is significant. Local recurrence after treatment for extremity soft tissue sarcoma approaches one out of three patients. The median disease- free interval is 18 months, but can be quite remote.

Considerations for treatment of recurrent disease are similar to primary disease and can include reexcision, chemo- therapy, and radiation, often with similar success to the primary tumors or extremity sarcomas. Recurrence of visceral and retroperitoneal sarcomas often is unable to be completely re-excised.

GIST is the most common mesenchymal tumor of the GI tract, and has gained significant interest in recent years. The discovery of the KI proto-oncogene mutation present in the majority of GIS tumors led to the ability to specifically target GIS tumors at a molecular level with the tyrosine kinase inhibitor imatinib mesylate (Gleevec; Novartis Pharmaceutical, Basel, Switzerland). This advancement marks a new era in treating solid tumors with specific molecular targeting.

Surgery remains of vital importance, but several studies have demonstrated improved disease-free and overall survival for patients at significant risk for recurrence who are treated adjuvantly with tyrosine kinase inhibitors, so this has become an important, disease-specific therapy
or these rare tumors.

How well did you know this?
1
Not at all
2
3
4
5
Perfectly
29
Q

A 26-year-old male presents with a painless mass in his left axilla that has been present for several months. Upon questioning, he endorses unexplained low-grade fevers, night sweats, and an unplanned weight loss of 10 lbs over the previous 3 months. On physical exam, he is noted to have a 2 cm firm, rubbery, non-tender mass in the left axilla.

Which of the following considerations is true for this patient?

A. Observation is warranted.

B. History of fever, chills, night sweats, and weight loss are considered A level symptoms.

C. Physical examination should include thorough exam of all accessible lymph node basins as well as potential sites of a primary malignancy.

D. Sexually transmitted diseases can be ruled out by
physical exam in younger patients.

E. Fine-needle aspiration (FNA) biopsy of suspicious lymph nodes in this patient is usually adequate or diagnosis.

A

C. Physical examination should include thorough exam of all accessible lymph node basins as well as potential sites of a primary malignancy.

In patients without a tissue diagnosis of malignancy, but concern for lymphoma, FNA is generally not considered to be adequate or diagnosis. FNA may be able to identify some cases of metastatic carcinoma (i.e., breast, lung, etc.) but is limited in hematologic malignancies (especially lymphoma) as lymph node architecture is a critical pathologic component to making these diagnoses.

FNA may be useful in searching or recurrent disease. As knowledge and use of molecular markers is expanded in the future, FNA may become a viable alternative to open biopsy of suspicious nodes. When lymphadenopathy
is generalized the largest, most suspicious, and accessible note is selected for biopsy. The diagnostic yield does vary by site with inguinal nodes having the lowest yield and supraclavicular nodes the highest. Careful and appropriate handling of tissue specimens is critical to allow pathologic diagnosis. Adequate tissue sampling is critical to allow assessment of nodal architecture. Specimens should be submitted to pathology dry or in saline, not formalin or other preservative, in order to allow flow cytometric and immunohistochemical studies to supplement traditional pathology.

The differential diagnosis or lymphadenopathy is extremely broad and includes malignancy (lymphoma, metastatic disease), infections (cat-scratch disease, HIV, mononucleosis, tuberculosis, etc.), autoimmune disorders, iatrogenic causes such as medications and unusual causes including sarcoidosis and Kawasaki’s disease.

A thorough history and physical can be used to help narrow this broad differential. In patients without concerning signs or symptoms suggestive o malignancy or other severe disease (e.g., B symptoms, etc.) a period of observation of four to six weeks is appropriate.

If the lymphadenopathy persists beyond this period of observation, further evaluation to include CBC and chest X-ray should be pursued.

Other testing to include serology or CMV, heterophile testing (monospot), PPD, HIV testing, RPR, and so on may be utilized based upon the history and physical and specific concerns.

Several non-specific laboratory tests reflecting inflammation to include ESR, CRP, and fibrinogen may be used, but do not typically help narrow the differential diagnosis. Lactate dehydrogenase (LDH) is often similarly non-specific though very high levels may suggest a lymphoid neoplasm. Ultimately, persistent lymphadenopathy without an obvious etiology warrants lymph node biopsy as discussed above.

Fever, chills, night sweats, and weight loss > 10% over 6 months are the classically described “B” symptoms associated with lymphoma and the presence of these findings should increase the suspicion of this diagnosis and prompt a more aggressive approach to diagnosis.

Careful physical exam should be an important part of the evaluation of every patient with lymphadenopathy. The differential diagnosis is highly dependent upon the presence of generalized versus isolated lymphadenopathy and the lymph node basin involved when localized. Examining sites of possible metastatic spread, or example breast, skin, oropharynx, and soon is important in narrowing the differential.

How well did you know this?
1
Not at all
2
3
4
5
Perfectly
30
Q

Regarding lymphoma, which of the following is true?

A. Surgeons play a major role in the treatment of lymphoma.

B. The hallmark of treatment is surgery and chemotherapy.

C. Staging or Hodgkin’s lymphoma is based upon a different staging system than is Non-Hodgkin’s lymphoma.

D. HIV infection has been shown to increase the incidence of lymphoma but other immunosuppressive states have not.

E. Mucosa-associated lymphoid tissue (MALT) lymphoma may be definitively treated with H. pylori eradication in its early stages.

A

E. Mucosa-associated lymphoid tissue (MALT) lymphoma may be definitively treated with H. pylori eradication in its early stages.

Lymphoma refers to a spectrum of diseases including Hodgkin’s and Non-Hodgkin’s (NHL) lymphomas with multiple subtypes within each group. The role of the surgeon in patients with lymphoma is limited.

By far, the most important role of the surgeon is in assisting with the diagnosis by means of lymph node biopsy. Once the diagnosis has been made, the treatment of lymphoma is overwhelmingly the role of chemotherapy and radiation.

Historically, surgery was more involved through the use of staging laparotomy, but this is now rarely indicated with the availability and use of PET/CT for staging purposes. Surgical intervention to include splenectomy is limited to significantly symptomatic situations such as anemia, thrombocytopenia, neutropenia and massive splenomegaly, and should be done only in close consultation with oncology.

Staging for lymphoma is based upon modifications of the original Ann Arbor classification, initially described for Hodgkin’s lymphoma, but now also used or NHL. Staging is based upon number and location of lymph node groups, extranodal involvement, and has modifiers to signify presence or absence of B symptoms.

Several risk factors for lymphoma have been identified. There has been a significant increase in incidence since the 1970s, which has been partially explained by the HIV epidemic. Other contributors to the increase include other infections (i.e. Helicobacter pylori-induced mucosal associated lymphoid tissue (MALT) lymphoma), autoimmune diseases, immunosuppression (as with organ transplant), environmental actors including pesticides, and aging of the population (NHL increases in incidence with age and peaks during the fifth through seventh decades).

Hodgkin’s lymphoma is more common in higher socioeconomic groups and less common in Asian populations.

MALT lymphoma has been shown to be dependent on Helicobacter pylori infection. Gastric inflammatory response to this infection stimulates the acquisition of genetic abnormalities and malignant transformation of B cells. In the early stages of this disease, H. pylori eradication can reverse the disease process in as many of 77% of patients. However, in later stages of the disease, further genetic injury to these malignant cells make the tumor resistant to bacterial eradication, so more aggressive therapy including chemotherapy and radiation is indicated. Like other forms of lymphoma, surgery is generally reserved or rare situations with complication such as life-threatening hemorrhage.

How well did you know this?
1
Not at all
2
3
4
5
Perfectly
31
Q

Following caustic injury to the skin with an alkaline agent, the effected area should initially be:

A. Treated with running water or saline for 30 minutes

B. Treated with running water or saline for 2 hours

C. Treated with a neutralizing agent

D. Treated with topical emollients and oral analgesics

A

B. Treated with running water or saline for 2 hours

The treatment for both types of injuries is based on neutralization of the inciting solution and starts with running distilled water or saline over the affected skin for at least 30 minutes for acidic solutions and 2 hours for alkaline injuries.

It should be noted that neutralizing agents do not offer a significant advantage over dilution with water, may delay treatment, and may worsen the injury due to the exothermic reaction that may occur.

The clinician observes and treats based on the degree of presentation. Many cases are successfully managed conservatively with topical emollients and oral analgesics, and most cases result in edema, erythema, and induration. If signs of deep second-degree burns develop, local wound care may include debridement, Silvadene, and protective petroleum gauze.

In severe cases, injury to the underlying vessels, bones, muscle, and tendon may occur, and these cases may be managed within 24 hours by liposuction through a small catheter and then saline injection.

Surgery is indicated for tissue necrosis, uncontrolled pain, or deep-tissue damage. Antibiotics should not be administered unless signs of infection are present. (See Schwartz 10th ed., p. 479.)

How well did you know this?
1
Not at all
2
3
4
5
Perfectly
32
Q

The treatment of a hydrofluoric acid skin burn is

A. Application of calcium carbonate gel

B. Irrigation with sodium bicarbonate

C. Injection of sodium bicarbonate

D. Local wound care only

A

A. Application of calcium carbonate gel

Injuries that have specific additional treatments include
hydrofluoride burns. Hydrofluoride is found in air conditioning cleaners and petroleum refineries.

Treatment of hydrofluoride burns should include topical or locally injected calcium gluconate to bind fluorine ions. Intra-arterial calcium gluconate can provide pain relief and preserves arteries from necrosis, whereas intravenous (IV) calcium repletes resorbed calcium stores.

Topical calcium carbonate gel and quaternary ammonium compounds detoxify fluoride ions. This mitigates the leaching of calcium and magnesium ions by the hydrofluoric acid from the affected tissues and prevents potentially severe hypocalcemia and hypomagnesemia that predispose to cardiac arrhythmias.

(See Schwartz 10th ed., pp. 479—480.)

How well did you know this?
1
Not at all
2
3
4
5
Perfectly
33
Q

The area most amenable to salvage by resuscitative and wound management techniques following thermal injury
is called the

A. Zone of hyperemia

B. Zone of coagulation

C. Zone of stasis

D. Zone of scalding

A

C. Zone of stasis

Exposure of the skin to thermal extremes disrupts its primary function as a barrier to heat loss, evaporation, and microbial invasion. The depth and extent of injury are dependent on the duration and temperature of the exposure. The pathophysiology and management are discussed elsewhere in this book.

Briefly, the epicenter of the injury undergoes a varying extent of necrosis (depending on the exposure), otherwise referred to as the zone of coagulation, which is surrounded by the zone of stasis, which has marginal perfusion and questionable viability. This is the area of tissue that is most amenable to salvage by appropriate resuscitative and wound management techniques, which would theoretically limit the extent of injury.

The outermost area of skin shows characteristics similar to other inflamed tissues and has been designated the zone of hyperemia. The degree of burn corresponds to histologic layers of the affected dermis and correlates with management and prognosis pertaining to timeline of healing and magnitude of scarring.

(See Schwartz 10th ed., p. 480.)

How well did you know this?
1
Not at all
2
3
4
5
Perfectly
34
Q

Tissue ischemia resulting in wounds that are characterized as a partial-thickness injury with a blister is considered

A. Stage 1

B. Stage 2

C. Stage 3

D. Stage 4

A

B. Stage 2

Tissue pressures that exceed the pressure of the microcirculation (30 mm Hg) result in tissue ischemia. Frequent or prolonged ischemic insults will ultimately result in tissue damage.

Areas of bony prominence are particularly prone to ischemia, the most common areas being ischial tuberosity (28%), trochanter (19%), sacrum (17%), and heel (9%). Tissue pressures can measure up to 300 mm Hg in the ischial region during sitting and 150 mm Hg over the sacrum while lying supine.

Muscle is more susceptible than skin to ischemic insult due to its relatively high metabolic demand.

Wounds are staged as follows:
stage 1, nonblanching erythema over intact skin;
stage 2, partial-thickness injury (epidermis or dermis)—
blister or crater;
stage 3, full-thickness injury extending
down to, but not including, fascia and without undermining of
adjacent tissue; and
stage 4, full-thickness skin injury with destruction or necrosis of muscle, bone, tendon, or joint
capsule.

(See Schwartz 10th ed., p. 482.)

How well did you know this?
1
Not at all
2
3
4
5
Perfectly
35
Q

The presence of sulfur granules in a draining wound should lead to the use of which of the following antibiotics?

A. Rifampin

B. Gentamicin

C. Penicillin

D. Amphotericin

A

C. Penicillin

Actinomycosis should be considered in the differential diagnosis of any acute, subacute, or chronic cutaneous swelling of the head and neck.

The cervicofacial form of Actinomycètes infection is the most common presentation, typically as an acute pyogenic infection in the submandibular or paramandibular area, but infection could be elsewhere in the mandibular and maxillary regions.

The primary skin infection may spread to adjacent structures such as the scalp, orbit, ears, and other areas.

Oral infection may spread to the hypopharynx, larynx, trachea, salivary glands, and sinuses. Actinomycosis can spread beyond boundaries of tissue planes and may also mimic chronic osteomyelitis.

Treatment consists of a combination of penicillin therapy and surgical debridement. Debulking and debriding infected tissue arising from sinus tracts and abscess cavities inhibit actinomycosis growth in most cases.

(See Schwartz 10th ed., p. 484.)

How well did you know this?
1
Not at all
2
3
4
5
Perfectly
36
Q

Initial treatment of nonpurulent, complicated cellulitis is

A. Vancomycin

B. ß-lactam

C. Linezolid

D. Clindamycin

A

B. ß-lactam

Treatment of nonpurulent, complicated cellulitis can begin with a ß-lactam, with methicillin-resistant Staphylococcus aureus (MRSA) coverage added if no response is observed.

Empiric MRSA coverage is warranted in all other complicated skin and subcutaneous infections.

Vancomycin is the mainstay of therapy, although it is inferior to ß-lactams for methicillin-sensitive S. aureus (MSSA) and has a relatively slow onset of efficacy in vitro. Linezolid, daptomycin, tigecycline, and telavancin are other FDA-approved alternatives for MRSA treatment.

Clindamycin is also approved for S. aureus; however, resistance may develop, and diarrhea can occur in up to 20% (Clostridium difficile related).

(See Schwartz 10th ed.,p.483.)

How well did you know this?
1
Not at all
2
3
4
5
Perfectly
37
Q

A 3-mm basal cell carcinoma (BCC) of the skin should be treated with

A. Biopsy and gross total excision

B. Dermatologic laser vaporization

C. Excision with 2- to 4-mm normal margin

D. Electrodesiccation

A

C. Excision with 2- to 4-mm normal margin

Basal cell carcinoma (BCC) arises from the basal layer of nonkeratinocytes and represents the most common tumor diagnosed in the United States. Annually it accounts for 25% of all diagnosed cancers and 75% of skin cancers. The primary risk factor for disease development is sun exposure (ultraviolet [UV] B rays more than UVA rays) particularly during adolescence; however, other factors include immune suppression (ie,
organ transplant recipients, human immunodeficiency virus [HIV]), chemical exposure, and ionizing radiation exposure.

BCC can also be a feature of inherited conditions such as xeroderma pigmentosa, unilateral basal cell nevus syndrome, and nevoid BCC syndrome. The natural behavior of BCC is one of local invasion rather than distant metastasis. Untreated BCC can result in significant morbidity. Thirty percent of cases are found on the nose, and bleeding, ulceration, and itching are
often part of the clinical presentation.

(See Schwartz 10th ed., p.486.)

How well did you know this?
1
Not at all
2
3
4
5
Perfectly
38
Q

Trichilemmal cysts

A. Are the most common type of cutaneous cysts

B. Are found between the forehead to nose |ip

C. Are typically found on the scalp of females

D. Occasionally develop bone, tooth, or nerve tissue

A

C. Are typically found on the scalp of females

There are three types of cutaneous cysts: epidermal, dermoid, and trichilemmal. All of these benign entities comprise epidermis that grows toward the center of the cyst, resulting in central accumulation of keratin to form a cyst. All clinically appear as a white, creamy substance-containing subcutaneous, thin-walled nodule.

Epidermal cysts are the most common cutaneous cyst and histologically characterized by mature epidermis complete with granular layer.

Trichilemmal cysts are the second most common lesion; they tend to form on the scalp of females, have a distinct odor after rupture, histologically lack a granular layer, and have an outer layer resembling the root sheath of a hair follicle.

Dermoid cysts are congenital, found between the forehead to nose tip, and contain squamous epithelium, eccrine glands, and pilosebaceous units, occasionally developing bone, tooth, or nerve tissue. The eyebrow is the most frequent site of presentation. These cysts are commonly asymptomatic but can become inflamed
and infected, thus necessitating incision and drainage.

After the acute phase subsides, the entire cyst should be removed to prevent recurrence.
(See Schwartz 10th ed., p. 486.)

How well did you know this?
1
Not at all
2
3
4
5
Perfectly
39
Q

More than half of patients treated for BCC will experience a recurrence within

A. 6 months

B. 1 years

C. 2 years

D. 3 years

A

D. 3 years

It is critical for each patient to have routine annual follow-up that includes full-body skin examinations. Sixty-six percent of recurrences develop within 3 years, and with a few exceptions occurring decades after initial treatment, the remaining recur within 5 years of initial treatment. A second primary BCC may develop after treatment and, in 40% of cases, presents within the first 3 years after treatment.

(See Schwartz 10th ed.,p. 487.)

How well did you know this?
1
Not at all
2
3
4
5
Perfectly
40
Q

The primary risk factor for the development of squamous cell carcinoma (SCC) is

A. UV exposure

B. Cigarette smoking

C. Chemical agents

D. Chronic, nonhealing wounds

A

A. UV exposure

Squamous cell carcinoma (SCC) is the second most common skin cancer, accounting for approximately 100,000 cases each year and generally afflicting individuals of lighter skin color.

The primary risk factor and driving force for the development of this common cancer is UV exposure; however, other risks include environmental factors such as chemical agents, physical agents (ionizing radiation), psoralen and UVA (PUVA), HPV-16 and -18 infections (immunosuppression), and smoking. Chronic nonhealing wounds, burn scars, and chronic dermatosis are other risk factors, and many darker skin individuals who develop SCC often have a history of one of these risk factors.

Heritable conditions such as xeroderma pigmentosum, epidermolysis bullosa, and oculocutaneous albinism
are predisposing risk factors.

(See Schwartz 10th ed., p. 487.)

How well did you know this?
1
Not at all
2
3
4
5
Perfectly
41
Q

In the ABCDE of melanoma, the D stands for diameter greater than

A. 2 mm

B. 4 mm

C. 6 mm

D. 8 mm

A

C. 6 mm

Melanoma most commonly manifests as cutaneous disease, and clinical characteristics include an Asymmetric outline, changing irregular Borders, Color variations, Diameter greater than 6 mm, and Elevation (ABCDE). Other key clinical characteristics include a pigmented lesion that has enlarged, ulcerated, or bled.

Amelanotic lesions appear as raised pink, purple, or normal-colored skin papules and are often diagnosed late.

(See Schwartz 10th ed., p. 488.)

How well did you know this?
1
Not at all
2
3
4
5
Perfectly
42
Q

The most common site of distant metastasis for melanoma is

A. Brain

B. Lung

C. Gastrointestinal tract

D. Distant skin

A

B. Lung

The most common sites of distant metastasis are the lungs and liver followed by the brain, gastrointestinal tract, distant skin, and subcutaneous tissue.

A limited subset of patients with small-volume, limited distant metastases to the brain, gastrointestinal tract, or distant skin will be cured with resection or gamma knife radiation.

Liver metastases are better dealt without surgical resection unless they arise from an ocular primary.

(See Schwartz 10th ed.,p. 491.)

How well did you know this?
1
Not at all
2
3
4
5
Perfectly
43
Q

The most common subtype of melanoma is

A. Lentigo maligna

B. Acral lentiginous

C. Superficial spreading

D. Nodular

A

C. Superficial spreading

Melanoma growth most commonly starts as a localized, radial growth phase followed by a vertical growth phase that determines metastatic risk.

The subtypes of melanoma include lentigo maligna, superficial spreading, acral lentiginous, mucosal, nodular, polypoid, desmoplastic, amelanotic, and soft tissue.

The most common subtype is superficial spreading, accounting for 70% of cases.

(See Schwartz 10th ed., p.488.)

How well did you know this?
1
Not at all
2
3
4
5
Perfectly
44
Q

Ocular melanoma

A. Exclusively metastasizes to the lungs

B. Exclusively metastasizes to the brain

C. Exclusively metastasizes to regional lymph nodes

D. Exclusively metastasizes to the liver

A

D. Exclusively metastasizes to the liver

Ocular melanoma is the most common noncutaneous disease site, and treatment includes photocoagulation, partial resection, radiation, or enucleation.

Ocular melanomas exclusively metastasize to the liver and not regional lymph nodes, and some patients benefit from liver resection.

(See Schwartz 10th ed.,p. 491.)

How well did you know this?
1
Not at all
2
3
4
5
Perfectly
45
Q

The following is NOT true in regard to Merkel cell carcinoma

A. It is commonly found in white men with a median age of 70 years.

B. It is characterized by a rapidly growing, flesh-colored papule.

C. Treatment should begin with examination of nodal
basins.

D. Recurrence is uncommon.

A

D. Recurrence is uncommon.

This is a rare and aggressive neuroendocrine tumor of the skin most commonly found in white men and diagnosed at a mean age of 70 years. Risk factors include UV radiation, PUVA, and immunosuppression.

Approximately one in three cases present on the face, with the remainder occurring on sun-exposed skin. A rapidly growing, flesh-colored papule or plaque characterizes the disease.

Regional lymph nodes are involved in 30% of patients, and 50% will develop systemic disease (skin, lymph nodes, Ever, lung, bone, brain).

There are no standardized diagnostic imaging studies for staging, but computed tomography (CT) of the chest, abdomen, and pelvis and octreotide scans may provide useful information when clinically indicated. After examining the entire skin for other lesions, treatment should begin by evaluating the nodal basins.

How well did you know this?
1
Not at all
2
3
4
5
Perfectly
46
Q

What is the most common melanoma in patients with
dark skin?

A. Nodular

B. Superficial spreading

C. Acral lentiginous

D. Lentigo maligna

A

C. Acral lentiginous

Nodular melanoma accounts for 15 to 30% of melanomas, and this variant is unique because it begins with a vertical growth phase that partly accounts for its worse prognosis.

Lentigo maligna is typically found in older individuals and primarily located in the head and neck region.

The acral lentiginous variant accounts for 29 to 72% of melanomas in dark-skinned individuals, is occasionally seen in Caucasians, and is found on palmar, plantar, and subungual surfaces.

(See Schwartz 10th ed.,p. 488.)

How well did you know this?
1
Not at all
2
3
4
5
Perfectly
47
Q

Not true about Kaposi sarcoma

A. Excision is the treatment of choice

B. Is predominantly found on the skin

C. Appears as rubbery, blue nodules

D. Is most often seen in patients in their fifth decade of life

A

A. Excision is the treatment of choice

Kaposi sarcoma is diagnosed after the fifth decade of life and predominantly found on the skin but can occur anywhere in the body. In North America, the Kaposi sarcoma herpes virus is transmitted via sexual and nonsexual routes and predominantly affects individuals with compromised immune systems such as those with HIV and transplant recipients on immune-suppressing medications.

Clinically, Kaposi sarcoma appears as multifocal, rubbery blue nodules. Treatment of acquired immunodeficiency syndrome (AIDS)-associated Kaposi sarcoma is with antiviral therapy, and many patients experience a dramatic treatment response. Those individuals who do not respond and have limited mucocutaneous disease may benefit from cryotherapy, photodynamic therapy, radiation therapy, intralesional injections, and topical therapy.

Surgical biopsy is important for disease diagnosis, but given the high local recurrence and the fact that Kaposi sarcoma represents more of a systemic rather than local disease, the benefit of surgery is limited and generally should not be pursued except for palliation.

(See Schwartz 10th ed., p. 492.)

How well did you know this?
1
Not at all
2
3
4
5
Perfectly
48
Q

The following is NOT a prognostic indicator for patients
with a sentinel node containing metastatic melanoma

A. Patient age

B. Site of metastasis

C. Number of positive nodes

D. Thickness, mitotic rate, and ulceration of primary
tumor

A

B. Site of metastasis

Melanoma is characterized according to the American Joint Committee on Cancer (AJCC) as localized disease (stage I and II), regional disease (stage III), or distant metastatic disease (stage IV). Overall tumor thickness, ulceration, and mitotic rate are the most important prognostic indicators of survival.

If a sentinel node contains metastatic melanoma, the number of positive nodes; thickness, mitotic rate, and ulceration of the primary tumor; and patient age determine prognosis.

With clinically positive nodes, the number of positive nodes, primary tumor ulceration, and patient age determine prognosis. The site of metastasis is strongly associated with prognosis for stage IV disease, and elevated lactate dehydrogenase (LDH) is associated with a worse prognosis.

(See Schwartz 10th ed., p. 488.)

How well did you know this?
1
Not at all
2
3
4
5
Perfectly
49
Q

A patient with a 5-mm deep melanoma of the thigh and
no clinically positive nodes should undergo which
procedure?

A. Resection of the primary only

B. Superficial femoral node resection

C. Superficial and deep femoral node resection

D. Resection of femoral and inguinal nodal basins

A

A. Resection of the primary only

Nonmetastatic in-transit disease should undergo excision to clear margins when feasible. However, disease not amenable to complete excision derives benefit from isolated limb perfusion (ILP) and isolated limb infusion (ILI) (Fig. 16-1). These two modalities are used to treat regional disease, and their purpose is to administer high doses of chemotherapy, commonly melphalan, to an affected limb while avoiding systemic drug toxicity.

ILI was shown to provide a 31% response rate in one study, while hyperthermic ILP provided a 63% complete response rate in an independent study.

(See Schwartz 10th ed., Figure 16-15, p. 491.)

How well did you know this?
1
Not at all
2
3
4
5
Perfectly
50
Q

A 65-year-old patient who spends winters in Florida
presents with a painless, ulcerated lesion on his right
cheek. The lesion has been present for 1 year. Physical
examination of the patient’s neck reveals no lymph node
enlargement. The most likely diagnosis is

A. Melanoma

B. BCC

C. SCC

D. Sebaceous cyst

A

B. BCC

The most common form of BCC (60%) is the nodular variant, characterized by raised, pearly pink papules and occasionally a depressed tumor center with raised borders giving the classic “rodent ulcer” appearance. This variant tends to develop in sun-exposed areas of individuals older than 60 years.

Superficial BCC accounts for 15% of BCC, is diagnosed at a mean age of 57 years, and typically appears on the trunk as a pink or erythematous plaque with a thin pearly border.

The infiltrative form appears on the head and neck in the late 60s with similar clinical appearance to the nodular variant.

An important variant to keep in mind is the pigmented variant of nodular BCC because this may be difficult to differentiate from nodular melanoma.

Other important subtypes include the morpheaform variant, accounting for 3% of cases and characterized by indistinct borders with a yellow hue, and fibroepithelioma of Pinkus. Histologic subtypes of BCC include nodular and micronodular (50%), superficial (15%), and infiltrative.

(See Schwartz 10th ed., p. 486.)

How well did you know this?
1
Not at all
2
3
4
5
Perfectly
51
Q

The chronic inflammatory disease presenting as painful subcutaneous nodules is

A. Pyoderma gangrenosum

B. Toxic epidermal necrolysis syndrome

C. Hidradenitis suppurativa

D. Steven-Johnson syndrome

A

C. Hidradenitis suppurativa

Hidradenitis suppurativa is a chronic inflammatory disease presenting as painful subcutaneous nodules. Patients experience appreciable physical, psychological, and economical hardship and decreased quality of life when compared with patients who suffer from other chronic dermatologic disease such as psoriasis and alopecia.

It is characterized by multiple abscesses, inter-networking sinus tracts, foul-smelling exudate from draining sinuses, inflammation in the dermis, both atrophic and hypertrophic scars, ulceration, and infection, which may extend deep into the fascia.

The diagnosis is made clinically without the need for
imaging or laboratory tests.

(See Schwartz 10th ed.,p. 467.)

How well did you know this?
1
Not at all
2
3
4
5
Perfectly
52
Q

Correct statements about toxic epidermal necrolysis (TEN) include all of the following EXCEPT

A. Toxic epidermal necrolysis is believed to be an immunologic problem.

B. Lesions are similar in appearance to partial thickness burns.

C. The process develops at the dermoepidermal junction.

D. Corticosteroid use is a primary part of therapy.

A

D. Corticosteroid use is a primary part of therapy.

These inflammatory diseases represent a spectrum of an
autoimmune reaction to stimuli such as drugs that result in structural defects in the epidermal-dermal junction.

The cutaneous manifestations of toxic epidermal necrolysis syndrome (TENS) follow a prodromal period reminiscent of an upper respiratory tract infection. A symmetrical macular eruption follows starting from the face and trunk and spreading to the extremities.

Typically, a Nikolsky sign develops in which lateral pressure causes the epidermis to detach from the
basal layer. The macular eruption evolves into blisters, causing an extensive superficial partial-thickness skin injury with exposed dermis.

(See Schwartz 10th ed., p. 477.)

How well did you know this?
1
Not at all
2
3
4
5
Perfectly
53
Q

The rare adenocarcinoma of the apocrine gland that of appears as a non-pigmented plaque is

A. Angiosarcoma

B. Extramammary Paget disease

C. Malignant fibrous histiocytoma

D. Dermatofibrosarcoma protuberans

A

B. Extramammary Paget disease

This rare adenocarcinoma of apocrine glands arises in perianal and axillary regions and in genitalia of men and women. Clinical presentation is that of erythematous or nonpigmented plaques with an eczema-like appearance that often persist after failed treatment from other therapies.

An important characteristic and one that the surgeon must be acutely aware of is the high incidence of concomitant other malignancies with this cutaneous
disease. Forty percent of cases are associated with primary gastrointestinal and genitourinary malignancies, and a diligent search should be made after a diagnosis of extramammary Paget disease is made.

Treatment is surgical resection with negative microscopic margins, and adjuvant radiation may provide additional locorégional control.

(See Schwartz 10th ed., p. 493.)

How well did you know this?
1
Not at all
2
3
4
5
Perfectly
54
Q

What are the two phases in the pathogenesis of melanoma?

A

1) Localized RADIAL growth phase

2) VERTICAL growth phase (determines metastatic risk)

How well did you know this?
1
Not at all
2
3
4
5
Perfectly
55
Q

What is the approach for any lesion suspicious for melanoma?

A

Suspicious lesion: Do a BIOPSY.

Small lesions (up to 1.5cm)

  • Excisional biopsy
  • 1 to 3mm margins
  • Deep margin up to subcutaneous fat
  • Orient biopsy with definitive wide excision in mind (parallel to LN)

Larger lesions

  • Incision biopsy of the most raised/pigmented area
  • Positive (incomplete) margins, but deep margin up to subcutaneous fat

Borderline lesions

  • Shave biopsy
  • Decreased index of suspicion for melanoma
Aesthetic areas (ie, face)
- Punch biopsy
How well did you know this?
1
Not at all
2
3
4
5
Perfectly
56
Q

What are the NCCN Recommendations for Staging Evaluation Based on Stage at Presentation for Melanoma?

A

PRIMARY TUMOR (No evidence of regional disease— Stage I & II)

  1. PE
  2. CXR
  3. LDH
  4. Sentinel Lymphadenectomy

LOCOREGIONAL DISEASE (Stage III)

  1. PE
  2. CXR
  3. LDH
  4. Sentinel lymphadenectomy
  5. WAB CT
  6. Chest CT
  7. Pelvic CT (if disease below waist)
  8. Neck CT (if head and neck area)
  9. Cranial MRI

DISTANT METS (Stage IV)

  1. PE
  2. CXR
  3. LDH
  4. WAB CT
  5. Chest CT
  6. Pelvic CT (if disease below waist)
  7. Neck CT (if head and neck area)
  8. Cranial MRI
  9. CLND (if clinically positive inguinofemoral nodes)
How well did you know this?
1
Not at all
2
3
4
5
Perfectly
57
Q

AJCC Staging for Soft Tissue Sarcoma of the Head and Neck?

A

Tx: Primary tumor cannot be assessed
T1: ≤2 cm
T2: >2 cm to ≤4 cm
T3: >4cm
T4: Invasion of adjoining structures
- T4a: Tumor with orbital invasion, skull base/dural invasion, invasion of central compartment viscera, involvement of fascial skeleton, invasion of pterygoids
- T4b: With brain parenchymal invasion, carotid artery encasement, prevertebral muscle invasion, or CNS involvement via perineural spread.

N0: No regional LN or unknown LN status
N1: Regional lymph node metastasis

M0: No distant metastasis
M1: Distant metastasis

GX: Grade cannot be assessed
G1: Total differentiation, mitotic count and necrosis score of 2 or 3
G2: Total differentiation, mitotic count and necrosis score of 4 or 5
G3: Total differentiation, mitotic count and necrosis score of 6, 7, or 8

Tumor Differentiation:

1: Sarcomas closely resembling normal adult mesenchymal tissue (e.g., lowgrade leiomyosarcoma)
2: Sarcomas for which histologic typing is certain (e.g., myxoid/round cell liposarcoma)
3: Embryonal and undifferentiated sarcomas, sarcomas of doubtful type, synovial sarcomas, soft tissue osteosarcoma, Ewing Sarcoma/primitive neuroectodermal tumor (PNET) of soft tissue

Mitotic Count:

1: 0-9 mitoses per 10 HPF
2: 10-19 mitoses per 10 HPF
3: ≥20 mitoses per 10 HPF

Tumor Necrosis

0: No necrosis
1: <50% tumor necrosis
2: ≥50% tumor necrosis

STAGING
Stage I (Low-grade)
Stage IA: T1 N0 M0 G1,GX
Stage IB: T2-T4 N0 M0 G1,GX

Stage II (Mod-high grade, ≤2 cm)
Stage II: T1 N0 M0 G2,G3

Stage III (Mod-high grade, >2cm)
Stage IIIA: T2 N0 M0 G2,G3
Stage IIIB: T3-T4 N0 MO G2,G3

Stage IV (Concurrent Mets)
Any T N1 M0 Any G
Any T Any N M1 Any G

How well did you know this?
1
Not at all
2
3
4
5
Perfectly
58
Q

AJCC Staging for Sarcoma of the Trunk and Extremities?

A
Tx: Primary tumor cannot be assessed
T1: ≤5 cm
T2: >5 cm to ≤10 cm
T3: >10 cm to ≤15 cm
T4: >15 cm

N0: No regional LN or unknown LN status
N1: Regional lymph node metastasis

M0: No distant metastasis
M1: Distant metastasis

GX: Grade cannot be assessed
G1: Total differentiation, mitotic count and necrosis score of 2 or 3
G2: Total differentiation, mitotic count and necrosis score of 4 or 5
G3: Total differentiation, mitotic count and necrosis score of 6, 7, or 8

Tumor Differentiation:

1: Sarcomas closely resembling normal adult mesenchymal tissue (e.g., lowgrade leiomyosarcoma)
2: Sarcomas for which histologic typing is certain (e.g., myxoid/round cell liposarcoma)
3: Embryonal and undifferentiated sarcomas, sarcomas of doubtful type, synovial sarcomas, soft tissue osteosarcoma, Ewing Sarcoma/primitive neuroectodermal tumor (PNET) of soft tissue

Mitotic Count:

1: 0-9 mitoses per 10 HPF
2: 10-19 mitoses per 10 HPF
3: ≥20 mitoses per 10 HPF

Tumor Necrosis

0: No necrosis
1: <50% tumor necrosis
2: ≥50% tumor necrosis

STAGING
Stage I (Low-grade)
Stage IA: T1 N0 M0 G1,GX
Stage IB: T2-T4 N0 M0 G1,GX

Stage II (Mod-high grade, ≤5 cm)
Stage II: T1 N0 M0 G2,G3

Stage III (Mod-high grade, >5cm)
Stage IIIA: T2 N0 M0 G2,G3
Stage IIIB: T3-T4 N0 MO G2,G3

Stage IV (Concurrent Mets)
Any T N1 M0 Any G
Any T Any N M1 Any G

How well did you know this?
1
Not at all
2
3
4
5
Perfectly
59
Q

AJCC Staging for Sarcoma of the Abdomen and Thoracic Visceral Organs?

A

Tx: Primary tumor cannot be assessed
T1: Organ confined
T2: Tumor extension into tissue beyond organ
- T2a: Invades serosa or visceral peritoneum
- T2b: Extension beyond serosa (mesentery)
T3: Invades another organ
T4: Multifocal involvement
- T4a: Multifocal (2 sites)
- T4b: Multifocal (3-5 sites)
- T4c: Multifocal (>5 sites)

N0: No regional LN or unknown LN status
N1: Regional lymph node metastasis

M0: No distant metastasis
M1: Distant metastasis

GX: Grade cannot be assessed
G1: Total differentiation, mitotic count and necrosis score of 2 or 3
G2: Total differentiation, mitotic count and necrosis score of 4 or 5
G3: Total differentiation, mitotic count and necrosis score of 6, 7, or 8

Tumor Differentiation:

1: Sarcomas closely resembling normal adult mesenchymal tissue (e.g., lowgrade leiomyosarcoma)
2: Sarcomas for which histologic typing is certain (e.g., myxoid/round cell liposarcoma)
3: Embryonal and undifferentiated sarcomas, sarcomas of doubtful type, synovial sarcomas, soft tissue osteosarcoma, Ewing Sarcoma/primitive neuroectodermal tumor (PNET) of soft tissue

Mitotic Count:

1: 0-9 mitoses per 10 HPF
2: 10-19 mitoses per 10 HPF
3: ≥20 mitoses per 10 HPF

Tumor Necrosis

0: No necrosis
1: <50% tumor necrosis
2: ≥50% tumor necrosis

How well did you know this?
1
Not at all
2
3
4
5
Perfectly
60
Q

AJCC Staging for Gastrointestinal Stromal Tumor (GIST)?

A
Tx: Primary tumor cannot be assessed
T0: No evidence of primary tumor
T1: ≤2 cm
T2: >2 cm to ≤5 cm
T3: >5 cm to ≤10 cm
T4: >10 cm

N0: No regional LN or unknown LN status
N1: Regional lymph node metastasis

M0: No distant metastasis
M1: Distant metastasis

Grading for GIST is based on mitotic rate
Low: ≤5 mitoses per 5mm^2, or per 50hpf
High: >5 mitoses per 5mm^2, or per 50hpf

Staging
Gastric GIST (Also: Omentum)
Stage IA: T1-T2 N0 M0, Low MR
Stage IB: T3 N0 M0, Low MR

Stage II: T1-T2 N0 M0, High MR
T4 N0 M0, Low MR

Stage IIIA: T3 N0 M0, High MR
Stage IIIB: T4 N0 M0, High MR

Stage IV: Any T, N1 M0, Any MR
Any T, Any N, M1, Any MR

Small Intestinal GIST (Also: Esophagus, colorectal, mesenteric, peritoneal)
Stage I: T1-T2 N0 M0, Low MR

Stage II: T3 N0 M0, Low MR

Stage IIIA: T1 N0 M0, High MR
T4 N0 M0, Low MR

Stage IIIB: T2-T4 N0 M0, High MR

Stage IV: Any T, N1 M0, Any rate
Any T, Any N, M1, Any rate

How well did you know this?
1
Not at all
2
3
4
5
Perfectly
61
Q

AJCC Staging for Sarcoma of the Retroperitoneum?

A
Tx: Primary tumor cannot be assessed
T1: ≤5 cm
T2: >5 cm to ≤10 cm
T3: >10 cm to ≤15 cm
T4: >15 cm

N0: No regional LN or unknown LN status
N1: Regional lymph node metastasis

M0: No distant metastasis
M1: Distant metastasis

GX: Grade cannot be assessed
G1: Total differentiation, mitotic count and necrosis score of 2 or 3
G2: Total differentiation, mitotic count and necrosis score of 4 or 5
G3: Total differentiation, mitotic count and necrosis score of 6, 7, or 8

Tumor Differentiation:

1: Sarcomas closely resembling normal adult mesenchymal tissue (e.g., lowgrade leiomyosarcoma)
2: Sarcomas for which histologic typing is certain (e.g., myxoid/round cell liposarcoma)
3: Embryonal and undifferentiated sarcomas, sarcomas of doubtful type, synovial sarcomas, soft tissue osteosarcoma, Ewing Sarcoma/primitive neuroectodermal tumor (PNET) of soft tissue

Mitotic Count:

1: 0-9 mitoses per 10 HPF
2: 10-19 mitoses per 10 HPF
3: ≥20 mitoses per 10 HPF

Tumor Necrosis

0: No necrosis
1: <50% tumor necrosis
2: ≥50% tumor necrosis

STAGING
Stage I (Low-grade)
Stage IA: T1 N0 M0 G1,GX
Stage IB: T2-T4 N0 M0 G1,GX

Stage II (Mod-high grade, ≤5 cm)
Stage II: T1 N0 M0 G2,G3

Stage III (Mod-high grade, >5cm)
Stage IIIA: T2 N0 M0 G2,G3
Stage IIIB: T3-T4 N0 MO G2,G3

Stage IV (Concurrent Mets)
Any T N1 M0 Any G
Any T Any N M1 Any G

How well did you know this?
1
Not at all
2
3
4
5
Perfectly
62
Q

Difference between UVA and UVB rays?

A

Environmental sources of radiation damage are typically from UV radiation. UVC rays are filtered by the ozone layer, so the only UV rays that humans typically encounter are UVA (320–400 nm) and UVB (290–320 nm).

The amount of exposure to UV radiation is dependent on seasonal, temporal, geographic and environmental variables.

Ninety-five percent of the UV rays that reach the earth’s surface are UVA rays. This radiation is less energetic (longer wavelength) than UVB rays and affects the cutaneous tissues differently. UVA waves penetrate deeper into the tissues, with 20% to 30% reaching the deep dermis.

UVB rays are mostly absorbed in the epidermis, with 70% reaching the stratum corneum, 20% reaching the deep epidermis, and only 10% reaching the papillary dermis.

Major chromophores in the cutaneous tissue include nucleic acids, aromatic amino acids, and melanin.

How well did you know this?
1
Not at all
2
3
4
5
Perfectly
63
Q

Short term effects of solar radiation?

A

The short-term effects of solar radiation include erythema and pigmentation. The resultant erythema peaks at 6 to 24 hours after exposure.

The pigmentation occurs differently for UVA and UVB rays.

Pigmentation occurs because of photooxidation of melanin by UVA radiation. Partial fading of this pigment change occurs within an hour after exposure, but with higher and repeated doses of UVA, stable residual pigmentation is observed.

UVB waves induce neomelanization, increasing the total amount of melanin in the epidermal tissues and resulting in an effect that is observable 72 hours after exposure. The increase in melanin as a result of UVB exposure serves as a protective mechanism to defend the nuclei of the basal keratinocytes from further radiation-induced damage by absorbing the high-energy radiation in future exposures.

How well did you know this?
1
Not at all
2
3
4
5
Perfectly
64
Q

Long term effects of UV radiation exposure?

A

Long-term effects of exposure to UV radiation can lead to chronic skin changes, such as irregular pigmentation, melasma, postinflammatory pigmentation, and actinic lentigines (sun spots).

Lysozyme, an enzyme secreted by cells of the immune system, typically inhibits the activity of collagenase and elastase, playing a role in turnover of the elastin and collagen network of the dermis.

Long-term exposure to UV radiation increases the activity of lysozyme, thus impairing the natural turnover of these fibers, resulting in a disorganized accumulation of elastin, and an increase in the ratio of type III to type I collagen.

This results in loss of firmness and resilience of the skin, leading to wrinkles and an aged appearance.

How well did you know this?
1
Not at all
2
3
4
5
Perfectly
65
Q

Pathophysiology of therapeutic radiation?

A

The various forms of radiation work to destroy the replicative potential of the target cells via damage to the nucleic acid structures in the cell. This is typically used to treat oncologic disease, but it can also be used to treat benign disease like eczema, psoriasis, and keloid scarring at relatively low exposures. While this goal is accomplished, surrounding tissues are also affected and damaged.

The most radiosensitive components of the cutaneous tissue are the basal keratinocytes, hair follicle stem cells, and melanocytes.

Exposure to this intense radiation results in disorganized, uncontrolled cell death, leading to the release of reactive oxygen species and further damage and inflammation to the surrounding cellular network. Damage to the basal keratinocytes and fibroblasts hinders the replicative capacity of the epidermis and dermis, respectively.

Acute skin changes to these structures manifest within weeks as erythema, edema, and alopecia. Permanent hyperpigmentation, tightening, thickening, and fibrosis of the skin become apparent as the tissue attempts to heal. In severe radiation injury, there can be complete loss of the epidermis, resulting in partial-thickness wounds and fibrinous exudate.

Reepithelialization typically occurs 14 days following initial injury, provided other variables affecting wound healing are optimized (bacterial colonization, nutrition.)

Long-term effects include compromise of the functional integrity of the skin secondary to thrombosis and necrosis of capillaries, hypovascularity, telangiectasia, ulceration, fibrosis, poor wound healing, and infection. These can present weeks to years after exposure.

How well did you know this?
1
Not at all
2
3
4
5
Perfectly
66
Q

Treatment of minor radiation injury?

A

Treatment of minor radiation injury includes skin moisturizers and local wound care when appropriate. Severe radiation injury may warrant surgical excision and reconstruction with free-tissue transfer from a part of the body unaffected by radiation.

67
Q

Recommended antibiotic prophylaxis for a human bite, and alternatives?

A

Amoxicillin/Clavulanate x 3-7 days

Alternatives: Doxycycline or Clindamycin with Ciprofloxacin

68
Q

AJCC Clinical Staging for Cutaneous Melanoma?

A

TX: Primary tumor thickness cannot be assessed (diagnosis by curettage)

T0: No evidence of primary tumor

Tis: Melanoma in situ

T1: ≤1 mm, unknown/unspecified

  • T1a <0.8mm, without ulceration
  • T1b <0.8mm, with ulceration
    0. 8-1.0mm, with/without ulceration

T2: >1.0–2.0mm, unknown/unspecified

  • T2a >1.0–2.0mm, without ulceration
  • T2b >1.0–2.0mm, with ulceration

T3: >2.0-4.0mm, unknown/unspecified

  • T3a >2.0-4.0mm without ulceration
  • T3b >2.0-4.0mm with ulceration

T4: >4.0mm, unknown/unspecified

  • T4a >4.0mm, without ulceration
  • T4b >4.0mm, with ulceration

NX: Regional nodes not assessed (SLNB not done, LN previously removed)
*Exception: when there are no clinically detected regional mets in a pT1 cM0 melanoma, assign cN0 instead of pNX

N0: No regional mets detected

N1: One tumor involved node, or in-transit, satellite and/or microsatellite mets with no tumor-involved nodes)

  • N1a one clinically occult (ie, by SLN biopsy)
  • N1b one clinically detected
  • N1c No regional lymph node disease (+in-transit, satellite and/or microsatellite metastases)

N2: Two or three tumor-involved nodes, or in-transit, satellite, and/or microsatellite metastases with one tumor-involved node

  • N2a Two or three clinically occult (ie by SLN biopsy)
  • N2b Two or three, at least one of which was clinically detected
  • N2c One clinically occult or clinically detected (+in-transit, satellite and/or microsatellite metastases)

N3: Four or more tumor-involved nodes or in-transit, satellite, and/or microsatellite metastases with two or more tumor-involved nodes, or any number of matted nodes without or with in-transit, satellite, and/or microsatellite metastases

  • N3a Four or more clinically occult (ie, detected by SLN biopsy)
  • N3b Four or more, at least one of which was clinically detected, or presence of any number of matted nodes
  • N3c Two or more clinically occult or clinically detected and/or presence of any number of matted nodes (+in-transit, satellite and/or microsatellite metastases)

M0: No evidence of distant mets

M1: Evidence of distant mets

  • M1a Distant mets to skin, soft tissue including muscle, and/or nonregional lymph node
  • -M1a (0) Normal LDH
  • -M1a (1) Elevated LDH
  • M1b Distant metastasis to lung with or without M1a sites of disease
  • -M1b (0) Normal LDH
  • -M1b (1) Elevated LDH
  • M1c Distant metastasis to non-CNS visceral sites with or without M1a or M1b sites of disease
  • -M1c (0) Normal LDH
  • -M1c (1) Elevated
  • M1d Distant metastasis to CNS with or without M1a, M1b, or M1c sites of disease
  • -M1d (0) Normal LDH
  • -M1d (1) Elevated LDH

Stage 0: Tis N0 M0

Localized Disease with No Evidence of Metastases:
Stage IA: T1a N0 M0
Stage IB: T1b-T2a N0 M0
Stage IIA: T2b-T3a N0 M0
Stage IIB: T3b-T4a N0 M0
Stage IIC: T4b N0 M0

Regional Disease
Stage III: Any T, Tis ≥N1 M0

Distant Metastatic Disease
Stage IV: Any T, Any N, M1

69
Q

Risk factors for Melanoma?

A

GENETIC

  1. Skin type
  2. Personal history of prior melanoma
  3. Multiple atypical/dysplastic nevi
  4. Positive family history of melanoma
  5. Inherited genetic mutations

ENVIRONMENTAL

  1. Excess sun exposure
  2. UV-based artificial tanning
70
Q

Most important prognostic indicators of survival in melanoma?

A
  1. Overall tumor thickness
  2. Mitotic rate
  3. Ulceration
71
Q

NCCN Biopsy Recommendations for Cutaneous Melanoma? (How do you approach a suspicious pigmented lesion?)

A

Patients presenting with a suspicious pigmented lesion optimally should undergo an excisional biopsy (elliptical, punch or saucerization), preferably with 1- to 3-mm negative margins.

The orientation of the excisional biopsy should always be planned with definitive treatment in mind (eg, a longitudinal orientation in the extremities, parallel to lymphatics).

With the increasing use of lymphatic mapping and sentinel node biopsy, biopsies should also be planned so as not to interfere with this procedure. In this regard, wider margins for the initial diagnostic procedure should be avoided.

For other sensitive/aesthetic sites, or very large lesions, a full-thickness incisional or punch biopsy of the clinically thickest portion is acceptable.

If inadequate initial biopsy, a re-biopsy with narrow margin excision should be done.

For low-suspicion settings, a shave biopsy may be acceptable.

72
Q

Among patients with nodal metastasis (stage III) in melanoma, what are the most important predictors of survival?

A
  1. Clinical node status (palpable vs. nonpalpable)

2. Number of metastatic nodes

73
Q

What are in-transit mets in melanoma?

A

In-transit metastasis is defined as intralymphatic tumor in skin or subcutaneous tissue more than 2 cm from the primary tumor but not beyond the nearest regional lymph node basin.

The presence of microsatellites, clinically evident satellites, and/or regional intransit disease is all part of the biologic continuum of regional lymphatic involvement, and these are all associated with a prognosis similar to that of patients with clinically positive nodes.

This is recognized in the staging system with the designation of stage IIIC.

74
Q

Among patients with distant metastatic melanoma (Stage IV), what is the most significant predictor of outcome?

A

Among patients with distant metastatic melanoma (stage IV), the site of metastases is the most significant predictor of outcome.

The three risk categories recognized by the AJCC are skin, soft tissue, and remote nodes (M1a); visceral-pulmonary (M1b); and visceral-nonpulmonary (M1c).

Elevated lactate dehydrogenase (LDH), likely a surrogate for overall tumor burden, is also an independent predictor of poor outcome in patients with stage IV disease and has been incorporated into the AJCC staging system; patients with distant metastases to any site and elevated LDH are in the highest risk category (M1c).

75
Q

How do you work-up a patient with a confirmed diagnosis of cutaneous melanoma?

A

After the diagnosis of cutaneous melanoma has been confirmed, detailed personal and family history, including any personal history of prior melanoma or dysplastic nevi, should be obtained.

In the physical examination of patients with invasive melanoma, physicians should pay special attention to the locoregional area and lymph node drainage basin(s) of the established melanoma.

A complete dermatologic examination is recommended for all patients with newly diagnosed melanoma.

76
Q

Based on preliminary workup and clinical staging patients are stratified into one of six groups for further workup and treatment:

A
  • Stage 0 (melanoma in situ); or stage IA or IB with thickness 0.75 mm or less, regardless of other features (eg, ulceration, mitotic rate)
  • Stage IA with thickness 0.76 to 1.0 mm, with no ulceration, and mitotic rate 0 per mm2
  • Stage IB with thickness 0.76 to 1.0 mm with ulceration or mitotic rate greater than or equal to 1 per mm2; or stage IB or II with thickness 1.0 mm thick, any feature (eg, with or without ulceration, any mitotic rate), and clinically negative nodes
  • Stage III with clinically detected (palpable) positive nodes, microscopic satellitosis (from assessment of the primary lesion), and/or in-transit disease
  • Stage IV (distant metastatic disease)
77
Q

How is SLNB performed for melanoma?

A

SLNB is almost always performed at the time of initial wide excision; the validity of performing this technique after definitive wide excision has not been extensively studied.

There is at least a theoretical concern that the relevant draining lymphatics could have been disturbed by the wide excision, especially if rotation flaps or skin grafts were used for reconstruction, degrading the accuracy of the SLNB procedure.

The technique for SLNB consists of:
1) preoperative dynamic lymphoscintigraphy,

2) intraoperative identification using isosulfan blue or methylene blue dye, and
3) a gamma probe to detect radiolabeled lymph nodes.

78
Q

Most common complications with SLNB for melanoma?

A

1) Wound dehiscence & infection
2) Seroma/hematoma
3) Lymphedema

79
Q

What are the indications for SLNB in melanoma?

A

1) T2 and higher (>1-4mm depth)
2) Thin melanoma with adverse features (>0.75mm, >1 mitosis per mm, ulcerated)

SLNB provides prognostic info and is therapeutic for low volume disease.

The incidence of regional mets increases significantly:
<1mm = ≤5%
>4mm = 35-40%

And also, there is at least a 10% risk of clinically occult nodal metastasis.

80
Q

Management of Stage 0, I, II Melanoma?

A

Stage 0, I, II
1) PE of primary site, regional lymphatic pathways, lymph node basin, and skin.

2) Nodal basin UTZ for equivocal regional PE prior to SLNB.

3) SLNB:
The NCCN Melanoma Panel does not recommend SLNB for patients with in situ melanoma (stage 0).
For patients with stage IA melanomas that are 0.76 to 1.0 mm thick without ulceration, and with mitotic rate 0 per mm2, SLNB should be considered in the appropriate clinical context.
SLNB should generally be discussed and offered for patients with higher- risk stage IB (>1 mm thick or 0.76–1.0 mm thick with ulceration or mitotic rate ≥1 per mm2) or stage II melanoma.

3) Wide-excision alone (in general).

81
Q

Management of Stage III Melanoma?

A

Stage III
Sentinel Node-Positive
1) Cross-sectional imaging as warranted by symptoms.

Clinically Node-Positive
1) Confirm suspicion with FNA, or core/incisional/excisional biopsy for enlarged lymph nodes.
If non-diagnostic, excisional biopsy, planned with therapeutic LN dissection in mind, is appropriate.

In-Transit Nodes
1) Confirm suspicion with FNA, or core/incisional/excisional biopsy for enlarged lymph nodes.
If non-diagnostic, excisional biopsy, planned with therapeutic LN dissection in mind, is appropriate.

2) Cross-sectional imaging as warranted by symptoms.

SLNB may be considered for patients with resectable solitary in-transit stage III disease (category 2B recommendation). However, while SLNB may be a useful staging tool, its impact on the OS of these patients remains unclear.

Since patients with stage IIIC have an appreciable risk of symptomatic CNS recurrence, and symptomatic CNS metastasis are associated with significant morbidity and poor survival, baseline CNS imaging should be considered in these high-risk patients.

82
Q

Management of Stage IV Melanoma?

A

1) Confirm suspicion of stage IV metastatic disease with FNA or CNB, incisional, excisional biopsy of the metastases.
2) Genetic analyses (BRAF or KIT mutation status are appropriate for patients being considered for treatment with targeted therapy, or if mutational status is relevant to eligibility for participation in a clinical trial. To ensure that adequate metastatic material is available for mutational analysis, biopsy (core, excisional, or incisional) is preferred if initial therapy is to be systemic and archival tissue is not available. However, the panel also recognized that brain metastases are typically treated without histologic confirmation.
3) Baseline chest/abdominal/pelvic CT.
4) Because patients with metastatic melanoma have a high incidence of brain metastases, brain MRI or CT scan with contrast should be performed at presentation with stage IV disease. Brain MRI is also recommended if patients have even minimal symptoms or physical findings suggestive of CNS involvement, or if results of imaging would affect decisions about treatment.
5) Serum LDH

83
Q

Clinical and surgical margins for wide excision of melanoma?

A

In-situ (Tis)
0.5 to 1cm margin (+ layer of subcutaneous tissue)

Melanomas ≤1.0mm thick (T1)
1cm margin (+ layer of subcutaneous tissue)

Melanomas 1.01 to 2mm thick (T2)
1-2cm margin

Melanomas >2mm thick (T3 & T4)
2cm margins

1- to 2-cm margins might be
acceptable in anatomically difficult areas where a full 2-cm margin would be difficult to achieve.

Depth is up to superficial fascia, but not through deep fascia.

For sole of the foot, excise down to the plantar fascia.

84
Q

What are the NCCN Recommendations for CLND in Melanoma?

A

If the sentinel node is negative, regional lymph node dissection is not indicated.

For patients with stage III disease based on a positive SLN, a CLND of the involved nodal basin should be discussed and offered, in the context of all of the points raised above, including the probability of a positive NSLN, the prognostic value of the NSLN status, the morbidity of the procedure, and the fact that one prospective randomized controlled trial has shown no benefit in any clinically relevant endpoint.

The impact of CLND on plans for adjuvant therapy or clinical trial enrollment should also be considered.

Patients presenting with clinically positive nodes without radiologic evidence of distant metastases should undergo wide excision of the primary site (if present) and CLND of the involved nodal basin.

In the setting of inguinal lymphadenopathy, a pelvic dissection is recommended if the PET/CT or pelvic CT scan reveals iliac and/or obturator lymph node involvement (category 2A) or if a positive Cloquet’s lymph node is found on intraoperative frozen section (category 2B). Pelvic dissection also should be considered for clinically positive inguinal-femoral nodes or if three or more inguinofemoral nodes are involved (category 2B).

For primary lesions in the head and neck with clinically or microscopically positive lymph nodes in the parotid gland, a superficial parotidectomy alone is insufficient and the panel recommends appropriate neck dissection of the draining nodal basins.

85
Q

NCCN Recommendations for adjuvant RT in melanoma?

A

Most patients with in situ or early-stage melanoma will be cured by primary excision alone. However, patients with desmoplastic melanomas, especially those with extensive neurotropism, are at high risk for local recurrence, especially if margins are suboptimal. Adjuvant radiation following surgery may be considered to improve local control.

Adjuvant RT may be considered for select patients with clinically positive nodes and features predicting a high risk of nodal basin relapse.

86
Q

NCCN Recommendations regarding the role of systemic therapy in melanoma?

A

Adjuvant treatment outside of a clinical trial is not recommended for patients with stage I/II disease, although the rationale for this recommendation varies across the NCCN Panel. There are no FDA- approved adjuvant immune checkpoint inhibitors or BRAF-targeted therapies for this group of patients. Although most of the trials to date did not include patients with stage I/II disease (Table 5), clinical trials are underway to define the role of adjuvant checkpoint inhibitors in high-risk stage II patients.

For patients with resected advanced melanoma, there have been a number of prospective randomized trials suggesting that immune checkpoint inhibitor and BRAF-targeted therapy are effective options for adjuvant treatment.

The most important factor to consider is the risk of recurrence and/or death from disease. Stage IIIA is the lowest risk group for which the NCCN Guidelines recommend considering adjuvant treatment.

87
Q

NCCN Recommendations for management of in-transit disease in melanoma?

A

Treatment in the context of a clinical trial is the preferred option for in- transit disease. For those with a single or a small number of resectable in- transit metastases, complete surgical excision with histologically negative margins is preferred, if feasible. In the patient undergoing curative resection of a solitary in-transit metastasis, SLNB can be considered (category 2B).

88
Q

NCCN Recommendations for Limited Metastatic Disease in Melanoma?

A

For limited metastatic disease, options include resection, if feasible, or systemic therapy. Observation is no longer a recommended option, even for patients with very limited stage IV disease, now that there are more effective active treatment options available. Systemic treatment should be followed by repeat scans to rule out the possibility that the disease is not more widespread, and to better select patients for surgical intervention.

89
Q

NCCN Recommendations for Disseminated Disease in Melanoma?

A

Disseminated disease can be managed by one or more of the following options, depending on the location of and extent of metastatic disease: clinical trial, systemic therapy, local treatment, or best supportive care (see the NCCN Guidelines for Palliative Care). For all systemic therapy options, consult the prescribing information for dosing recommendations. A number of options are available for systemic therapy.

For extracranial metastases, local treatment options may include intralesional injection with T-VEC, resection, or radiation. T-VEC can be injected into nodal or distant metastases to help with disease control, but the impact on survival is not known. It may be useful for patients with very limited stage IV disease, or in combination with other treatment modalities. Symptomatic extracranial metastases can be managed with palliative resection and/or radiation. Radiation can be used for palliation of visceral, bone, and CNS metastases.

Recommendations for First-line Systemic Therapy
For first-line therapy of unresectable or distant metastatic disease, recommended treatment options include immune checkpoint inhibitors, BRAF-targeted therapy for patients with an activating BRAF V600 mutation, or clinical trial.

Immune checkpoint inhibitor options in this setting include anti-PD-1 monotherapy with pembrolizumab (category 1) or nivolumab (category 1) or nivolumab/ipilimumab combination therapy (category 1). Immune checkpoint inhibitors have been shown to be effective regardless of BRAF mutation status. The NCCN Panel considers all recommended immune checkpoint inhibitor options appropriate for both BRAF mutant and BRAF wild-type metastatic disease.

90
Q

NCCN Recommendations for treating persistent disease or local scar recurrence in melanoma?

A

Persistent Disease or Local Scar Recurrence
The panel recognized the distinction between true local scar recurrence after inadequate initial excision (which most likely represents locally persistent disease) and local recurrence after adequate initial excision, (which likely represents dermal lymphatic disease appearing in proximity to the wide excision scar).732 In the former situation, defined by the presence of in situ and/or radial growth phase, the prognosis after re-excision is related to the microstaging of the recurrence, whereas the latter scenario is prognostically similar to recurrent regional disease.

For persistent disease or true local scar recurrence after inadequate primary therapy, a biopsy is required for confirmation. Guidelines for this biopsy should be the same as for primary tumors. The workup should be similar to that of the primary tumor based on microstaging characteristics. Re-excision to appropriate margins is recommended, with or without lymphatic mapping and SLNB according to primary tumor characteristics. Adjuvant treatment should be based on pathologic stage of the recurrence, and should be similar to that of primary tumors of equivalent stage.

91
Q

NCCN Recommendations in treating Local, Satellite, and/or In-Transit Recurrence in Melanoma?

A

Initial clinical recurrence should be confirmed pathologically whenever possible or if clinically indicated. Pathology should be confirmed by FNA cytology, if feasible, or core, incisional, or excisional biopsy. Local or satellite recurrences are in the deep dermis or subcutaneous fat within the melanoma scar or satellite metastasis adjacent to the melanoma scar. By definition they are recurrences after initial adequate wide excision, and lack in situ or radial growth phase. Tissue from the recurrence (preferred) or archival tissue should be assessed for mutation status if the patient is being considered for targeted therapy or enrollment in a clinical trial that includes mutation status as an eligibility criterion. Baseline imaging (CT and/or PET/CT or MRI) is recommended for staging and to evaluate specific signs or symptoms (category 2B).

Options for treatment of unresectable local, satellite, or in-transit recurrences include intralesional injection with T-VEC, ILP or ILIwith melphalan, or systemic therapy (as recommended for metastatic disease). The following are category 2B alternatives: intralesional injections with BCG, IFN alfa, or IL-2, topical imiquimod (for superficial dermal lesions), local ablation therapy, or RT.

After complete response to any of these modalities, options include participation in a clinical trial or observation. For those rendered free of disease by surgery, an additional adjuvant therapy option is high-dose IFN alfa (category 2B).

92
Q

NCCN Recommendations for Regional Nodal Recurrence in Melanoma?

A

For patients presenting with regional nodal recurrence, the clinical diagnosis should be confirmed by FNA (preferred) or core, incisional, or excisional biopsy. Tissue from the recurrence (preferred) or archival tissue should be assessed for mutation status if the patient is being considered for targeted therapy or enrollment in a clinical trial that includes mutation status as an eligibility criterion. Baseline imaging (CT and/or PET/CT or MRI) is recommended for staging and to evaluate specific signs or symptoms (category 2B).

For patients who have not undergone prior lymph node dissection or had an incomplete lymph node dissection, a CLND is advised. If the patient underwent a previous CLND, excision of the recurrence to negative margins is recommended if possible. After complete resection of nodal recurrence, options for adjuvant treatment include a clinical trial, observation, or, in patients who were not previously treated, high-dose or pegylated IFN alfa, high-dose ipilimumab (category 2B), or biochemotherapy (category 2B). Adjuvant radiation to the nodal basin may also be considered in selected high-risk patients based on size, location, and number of involved nodes, and/or macroscopic extranodal extension (category 2B). For patients with incompletely resected nodal recurrence, unresectable disease, or systemic disease, options include systemic therapy (preferred), clinical trial, palliative RT, intralesional injection with T- VEC, or best supportive care (see NCCN Guidelines for Palliative Care).

93
Q

NCCN Recommendations for Palliative Care in Metastatic Melanoma?

A

For patients presenting with distant recurrence, the workup and treatment options are similar to those outlined previously for patients presenting initially with stage IV metastatic disease.

94
Q

Genes and genetic syndromes involved in the pathogenesis of BCC?

A

GENES
- Sonic hedgehog signaling pathway (PTCH1 gene on chromosome 9q– underlying cause of nevoid BCC syndrome, and are present in 30-90% of sporadic BCCs.

  • UV-induced mutations in the tumor suppressor gene p53.

GENETIC SYNDROMES

  • Albinism (absent skin pigment)
  • Xeroderma pigmentosum (defects exist in UV light-induced unscheduled DNA repair)
95
Q

How do you approach a lesion suspicious for BCC?

A

On clinical presentation of the patient with a suspicious lesion, workup for BCC begins with a history and physical examination, with an emphasis on a complete skin examination. A full skin examination is recommended, because individuals with a skin cancer often have additional, concurrent precancers or cancers located at other, usually sun-exposed skin sites.

These individuals are also at increased risk of developing cutaneous melanoma.

A skin biopsy is then performed on any suspicious lesion. The biopsy should include deep reticular dermis if the lesion is suspected to be more than a superficial process. This procedure is preferred because an infiltrative histology may sometimes be present only at the deeper, advancing margins of a tumor, and superficial biopsies will frequently miss this component.

Imaging studies should be performed when extensive disease, such as bone involvement, perineural invasion, or deep soft tissue involvement, is suspected.

MRI is preferred over CT scan if perineural disease is suspected because of its higher sensitivity.

96
Q

Factors predictive of low risk of recurrence in BCC?

A

LOW RISK

  • Trunk and extremities (except hands, nail units, pretibia, ankles, feet) <20mm
  • Cheeks, forehead, scalp, neck pretibia <10mm
  • Well-defined borders
  • Primary tumor
  • Not immunocompromised
  • No site of prior RT
  • Nodular, superficial
  • No perineural involvement
97
Q

Factors predictive of high risk of recurrence in BCC?

A

HIGH RISK

  • Trunk and extremities (except hands, nail units, pretibia, ankles, feet) ≥20 mm
  • Cheeks, forehead, scalp, neck pretibia ≥10mm
  • Mask areas of face (central, eyelids, eyebrows, periorbital, nose, lips, chin, mandible, preauricular, postauricular skin/sulci, temple, ear, genitalia, hands, feet
  • Poorly defined borders
  • Recurrent
  • Immunosuppression
  • (+) site of prior RT
  • Aggressive growth pattern
  • (+) perineural involvement
98
Q

Most common type of skin cancer?

A

Basal cell carcinoma (80% of all skin cancers)

99
Q

Most common type of BCC?

A
Nodular BCC (60%)
- Classic domed, pearly papule with surface telangiectasia (rodent ulcer)
100
Q

Treatment options for BCC?

A

1) Surgical excision with postoperative margin assessment (>98% 5-year disease-free rates)
- Low-risk (well-circumscribed, <2cm): 4mm clinical margins

2) Mohs Micrographic Surgery or Excision with Intraoperative Frozen Section Assessment
- Preferred for high-risk BCC
- Intraop analysis of 100% of excision margin
- Alternative: Excision with Complete Circumferential Peripheral and Deep Margin Assessment (CCPDMA)

3) Curettage and Electrodessication
- For low-risk tumors except areas with terminal hair growth (tumor may extend down follicular structures)
- Alternately scraping away tumor tissue with a curette down to a firm layer of normal dermis and denaturing the area by electrodessication.
- Up to 3 cycles may be performed in a session.
- No histologic margin assessment.
- If subcutaneous layer is reached during C&E, do surgical excision.

4) Radiation Therapy
- For unresectable tumors, inferior cosmesis; advised only for >60 years old because of concerns about long-term sequelae.
- Contraindications: Genetic conditions predisposing to cancer and connective tissue diseases

5) Superficial Therapies
a. Topical (Imiquimod + 5-FU)
- Superficial BCC <2cm in non-immunocompromised patient
- 5days/week x 6 weeks (clearance rate >80%)

b. Cryosurgery
- Destroys tumor cells by freeze-thaw cycles
- Poorer cosmetic outcomes

c. Photodynamic Therapy
- Application of photosensitizing agent on skin followed by irradiation with a light source
- Methyl aminolevulinate (MAL) and 5-aminolevulinic acid (ALA)
- For premalignant or superficial low risk lesions on any location on the body.

In patients with superficial BCC, - PDT has similar efficacy as cryotherapy but much better cosmetic outcomes;
and

  • PDT, imiquimod, and fluorouracil have similar efficacy and cosmetic outcomes, although risk of recurrence may be somewhat higher with PDT versus imiquimod.

6) Intralesional Interferon
- Intralesional interferon alfa-2b can be effective for treating low-risk, superficial BCC

101
Q

NCCN Recommendations for the Management of BCC?

A

NCCN Recommendations:

1) LOW-RISK BCC
- C&E in areas without hair growth
- Standard excision if lesion can be excised with a 4mm clinical margins and with closure techniques such as linear closure, secondary intention, or skin graft
- RT for non-surgical candidates limited to those >60 years old.
- If margins are positive after excision, patients should receive adjuvant therapy. MMS, resection with CCPDMA with frozen or permanent section, or standard re-excision for area L regions (trunk and extremities, excluding pretibia, hands, feet, nail units, and ankle) are recommended, while radiation may be administered to non-surgical candidates.

2) HIGH-RISK BCC
- Standard excision using wider margins with linear or delayed repair, with standard re-excision
- MMS or resection with CCPDMA
- RT for non-surgical candidates

Patients treated with MMS or resection with CCPDMA should receive adjuvant therapy if clear margins cannot be achieved. Recommended
adjuvant therapy options include radiation and/or multidisciplinary consultation to consider systemic therapy with a hedgehog pathway inhibitor or treatment in the context of a clinical trial (vismodegib/ sonidegib).

Adjuvant RT is also recommended for patients with negative margins after surgery but with large nerve or extensive perineural involvement.

Due to the potential for skull involvement and intracranial extension, an MRI should be considered if large-nerve invasion is suspected for tumors on the head and neck.

If negative margins are not achieved after standard excision, patients should undergo MMS or resection with CCPDMA, or receive adjuvant RT. If residual disease is still present after adjuvant treatment, and further surgery and RT are contraindicated, clinicians should consider multidisciplinary consultation to determine whether the patient should be offered systemic treatment with a hedgehog pathway inhibitor or treatment in the context of a clinical trial.

102
Q

NCCN Recommendations for BCC Recurrence and Metastasis?

A

For the management of local tumor recurrence, the algorithm directs clinicians to follow the appropriate pathways for primary treatment.

Although the behavior of cutaneous BCC is characteristically indolent,
the disease does occasionally metastasize to distant sites. Whenever possible, nodal or distant metastases should be treated with surgery with or without RT, and managed by a multidisciplinary tumor board.

The board should consider systemic therapy with a hedgehog pathway
inhibitor or treatment in the context of a clinical trial. FDA-approved hedgehog pathway inhibitors include vismodegib and sonidegib.

103
Q

NCCN Recommendations for Follow-Up of BCC patients?

A

The frequency of follow-up should be based on risk. In addition to patient education about sun protection and self-examination, patients should be monitored with regular physical exams including complete skin examination. Monitoring during the first 2 years is the most critical, and exams should occur at least every 6 to 12 months during this timeframe. If no further skin cancer develops in the first 2 years, then it may be appropriate to reduce exam frequency.

104
Q

AJCC Staging of SCC?

A

TX: Primary tumor cannot be assessed

Tis: Carcinoma in situ

T1: Tumor smaller than or equal to 2 cm in greatest dimension

T2: Tumor larger than 2 cm, but smaller than or equal to 4 cm in greatest dimension

T3: Tumor larger than 4 cm in maximum dimension or minor bone erosion or perineural invasion or deep invasion*

T4: Tumor with gross cortical bone/marrow, skull base invasion and/or skull base foramen invasion

  • T4a Tumor with gross cortical bone/marrow invasion
  • T4b Tumor with skull base invasion and/or skull base foramen involvement

*Deep invasion is defined as invasion beyond the subcutaneous fat or >6 mm (as measured from the granular layer of adjacent normal epidermis to the base of the tumor); perineural invasion for T3 classification is defined as tumor cells within the nerve sheath of a nerve lying deeper than the dermis or measuring 0.1 mm or larger in caliber, or presenting with clinical or radiographic involvement of named nerves without skull base invasion or transgression.

Clinical N
NX: Regional lymph nodes cannot be assessed

N0: No regional lymph node metastasis

N1: Metastasis in a single ipsilateral lymph node, 3 cm or smaller in greatest dimension and ENE(−)

N2: Metastasis in a single ipsilateral node larger than 3 cm but not larger than 6 cm in greatest dimension and ENE(−);
or metastases in multiple ipsilateral lymph nodes, none larger than 6 cm in greatest dimension and ENE(−);
or in bilateral or contralateral lymph nodes, none larger than 6 cm in greatest dimension and ENE(−)

  • N2a Metastasis in a single ipsilateral node larger than 3 cm but not larger than 6 cm in greatest dimension and ENE(−)
  • N2b Metastases in multiple ipsilateral nodes, none larger than 6 cm in greatest dimension and ENE(−)
  • N2c Metastases in bilateral or contralateral lymph nodes, none larger than 6 cm in greatest dimension and ENE(−)

N3: Metastasis in a lymph node larger than 6 cm in greatest dimension and ENE(−);
or metastasis in any node(s) and clinically overt ENE [ENE(+)]
- N3a Metastasis in a lymph node larger than 6 cm in greatest dimension and
ENE(−)
- N3b Metastasis in any node(s) and ENE (+)

M0: No distant metastasis
M1: Distant metastasis

GX: Grade cannot be assessed G1: Well differentiated
G2: Moderately differentiated G3: Poorly differentiated
G4 Undifferentiated

Stage 0: Tis N0 M0
Stage I: T1 N0 M0
Stage II: T2 N0 M0
Stage III: T3 N0 M0
                T1-3 N1 M0
Stage IV: T1-3 N2 M0
                Any T, N3, M0
                T4, Any N, M0
                Any T, Any N, M1
105
Q

Risk factors for SCC?

A

1) Sun exposure/tanning
2) Scars or chronic wounds/trauma/burns (Marjolin ulcer)
3) Actinic keratoses (sun-induced precancerous lesions)
4) Bowen’s disease (SCC in situ)
5) Genetic syndromes: Albinism, xeroderma pigmentosum
6) Immunosuppression

106
Q

Approach to lesions suspicious for SCC?

A

1) Hx and PE, complete skin and regional LN exam (also at increased risk for other cancers such as melanoma)
2) Biopsy (include deep reticular dermis)
3) Imaging studies when extensive disease is suspected. MRI with contrast is preferred over CT if perineural disease or deep soft tissue involvement is suspected. If bone disease is expected, CT with contrast is preferred unless contraindicated.
4) Palpable LN should prompt an FNA or core biopsy.
5) Risk stratification of the primary tumor to determine treatment plan and follow up.

107
Q

Risk stratification for SCC?

A

LOW RISK

  • trunk and extremities (excluding hands, nail units, pretibia, ankles, and feet) <20mm
  • cheeks, forehead, scalp, neck, and pretibia <10mm
  • Well-defined borders
  • Primary tumor
  • No immunosuppression
  • No previous RT site/inflammation
  • Slow growing tumor
  • No neurologic symptoms
  • Well or mod differentiated
  • Not any of these: Acantholytic (adenoid), adenosquamous (showing mucin production), desmoplastic, or metaplastic (carcinosarcomatous) subtypes
  • ≤6 mm and no invasion beyond subcutaneous fat
  • No perineural, lymphatic or vascular involvement

HIGH-RISK

  • trunk and extremities (excluding hands, nail units, pretibia, ankles, and feet) ≥20mm
  • cheeks, forehead, scalp, neck, and pretibia ≥10mm
  • “mask areas” of face (central face, eyelids, eyebrows, periorbital, nose, lips [cutaneous and vermilion], chin, mandible, preauricular and postauricular skin/sulci, temple, and ear), genitalia, hands, and feet
  • Poorly- defined borders
  • Recurrent tumor
  • (+) Immunosuppression
  • (+) previous RT site/inflammation
  • Rapidly growing tumor
  • (+) Neurologic symptoms
  • Poorly differentiated
  • (+) Acantholytic (adenoid), adenosquamous (showing mucin production), desmoplastic, or metaplastic (carcinosarcomatous) subtypes
  • > 6 mm or with (+) invasion beyond subcutaneous fat
  • (+) Perineural, lymphatic or vascular involvement
108
Q

NCCN Recommendations for treating local SCC?

A

LOW-RISK
1) C&E in areas without hair growth (ie, excluding terminal hair-bearing regions, such as the scalp, pubic and axillary regions, and beard area in men), provided that the treatment be changed to excision if the adipose tissue is reached;

2) standard excision if the lesion can be excised with 4mm (<2cm lesions) to 6-mm (>2cm lesions) clinical margins and repaired with linear closure, secondary intention healing, or skin graft; and
3) RT for non-surgical candidates, generally limited to those older than 60 years of age because of risk of long-term toxicity.

If margins are positive after excision, patients should receive additional therapy. MMS, resection with CCPDMA with frozen or permanent section, or standard re-excision for area L regions (trunk and extremities, excluding pretibia, hands, feet, nail units, and ankle) are recommended, while radiation may be administered to non-surgical candidates.

HIGH-RISK
1) standard excision, using wider margins with linear or delayed repair; 
(4mm for <1cm;
6mm for 1-1.9cm;
9mm for >2cm)

2) MMS or resection with CCPDMA with frozen or permanent section; and
3) RT for non- surgical candidates.

Similarly, patients treated with MMS or resection with CCPDMA as first- line therapy should receive subsequent therapy if clear margins cannot be achieved. Recommended options include radiation (of primary site) and/or multidisciplinary consultation to consider chemoradiation or clinical trial. Appropriate chemotherapy agents can be found in the NCCN Guidelines for Head and Neck Cancers. If invasion into parotid fascia is discovered during MMS or resection with CCPDMA, then superficial parotidectomy is indicated.

Adjuvant RT to the primary site is recommended for patients with negative margins after surgery if there is large nerve or extensive perineural involvement. Due to the potential for skull involvement and intracranial extension, an MRI with contrast of the area of interest should be considered if large-nerve invasion is suspected for tumors on the head and neck.

For certain high-risk SCCs, SLNB mapping may be considered. A systematic review of 692 patients with SCC reported positive sentinel nodes in 24% and 21% of anogenital and non-anogenital patients, respectively.240 The survival benefit of and indication for SLNB remains unclear.

109
Q

Potential premalignant lesions?

A

1) Keratoacanthomas
- can mimic SCC/BCC
- grow rapidly & involute
- still warrant biopsy.

2) Actinic (Solar) Keratosis and Cutaneous Horns
- Premalignant lesions in sun-exposed areas of fair-skinned patients
- High risk of developing multiple primary cSCCs.
- Atypical clinical appearance or lack of response to therapy warrants biopsy.
- Tx options:
- – Topical 5-FU
- – Topical Imiquimod
- – Topical Ingenol mebutate
- – PDT
- – Cryotherapy
- – C&E

3) Actinic keratosis on the lip (Actinic cheilitis)
- Tx options:
- – Ablative laser vermillionectomy

4) Bowen Disease
- Slow growing erythematous plaque with scaling or crusting surface
- Sun exposure, immunosuppression, HIV
- Cutaneous SCC in situ
- 10% progress to invasive SCC
- Tx: Complete excision with negative margins.

5) Dysplastic Nevi
- Increased risk to develop MELANOMA.
- Red-brown, scalloped edges, variegated pigmentation, >6mm
- Trunk, non-exposed areas.

110
Q

Management of Merkel Cell Carcinoma?

A

Merkel cell carcinoma

  • Cutaneous neuroendocrine tumor from the neuroectoderm
  • Rapidly-growing, pink to red-blue violaceous firm nodule, in elderly patients
  • Associated with POLYOMA Virus
  • Tx:
    1) Wide excision with histologic confirmation of negative margins
    • <2cm tumor = 1cm margin
    • > 2cm tumor = 2cm margin
  • LV invasion is a major prognostic factor.
  • Poor overall survival (50-80% mort) with high recurrence.

2) For clinically node-negative MCC: Do SLNB with selective LN dissection!
3) Adjuvant radiotherapy improves locoregional control (fairly radio/chemosensitive tumor)

IHC:

  • Pancytokeratin AEI/AE3
  • Cytokeratin 20
  • Chromogranin A
111
Q

Management of Dermatofibrosarcoma protuberans?

A

Dermatofibrosarcoma protuberans

  • Rare, low-grade malignant CA of dermis
  • Most common DERMAL Sarcoma
  • Nodular, plaque-like, violaceous
  • Slow-growing, trunk and proximal extremities
  • Middle-aged adults
  • High local recurrence rates, but minimal metastatic potential.
  • Dx: Punch biopsy (spindle cells in dermis with storiform architecture, decreased mitotic rate, with fingerlike extensions at the border)
  • IHC: CD-34 expression.
  • Tx: Wide excision with 3-5cm margins that are pathologically negative.
  • Other tx options:
    1) Imatinib: Only for shown chromose 17-22 translocation

2) RT: Only for UNRESECTABLE disease/close margins.

112
Q

General management of thermal and caustic injury to skin?

A

Tx: Neutralize inciting solution in saline/distilled H2O!

ACIDIC Solution: 30mins
ALKALINE Solution: 2hours

113
Q

Management of Hydrofluoride burns?

A

1) Topical/locally injected calcium gluconate (binds fluoride ions)

2) Topical calcium carbonate gel + quaternary ammonium compounds
(detoxifies fluoride ions)

114
Q

Injury zones?

A

1) Zone of coagulation
- EPICENTER of injury

2) Zone of stasis
- Most amenable to salvage
- Marginal perfusion, questionable viability

3) Zone of hyperemia

115
Q

Management of Actinomycosis?

A

ACTINOMYCOSIS

  • most common: cervicofacial form
  • acute pyogenic infection in submandibular or paramandibular area
  • should be considered in any acute, subacute or chronic cutaneous swelling of the H&N
  • can spread beyond boundaries of tissue planes, may mimic chronic osteomyelitis
  • Tx: PENICILLIN + Surgical debridement
116
Q

Management of Cellulitis?

A

Treat with Beta-lactam antibotics (+MRSA Coverage if no response) for nonpurulent, complicated cellulitis

117
Q

Management of Kaposi Sarcoma?

A

Kaposi Sarcoma

  • After 5th decade of life
  • Mostly in skin
  • HIV & transplant patients
  • Multifocal rubbery blue nodules
  • Tx:
  • –AIDS associated Kaposi Sarcoma: Antiviral
  • – Cryotherapy, photodynamic tx, radiation tx, topical, intralesional
  • – Surgery is reserved for PALLIATION.
118
Q

Management of Hidradenitis Suppurativa?

A

Hidradenitis Suppurativa

  • Chronic inflammatory disease presenting as painful subcutaneous nodules (atrophy of sebaceous gland followed by inflammation of pilosebaceous unit causing hyperkeratosis and granuloma formation)
  • Foul-smelling exudate

RISK FACTORS

  • 3rd decade of life
  • Smoking and obesity
  • Genetic predisposition exacerbated by environmental factors (gamma-secretase gene mutation)

STAGES (HURLEY)

1: Single/multiple nodules or abscesses W/O sinus tracts or scarring
2: Abscesses recur and WITH sinus tract and scarring
3: Diffuse disease and interconnected sinus tracts and abscesses

  • Tx:
    Stage I/II: Clindamycin
    Advanced Stage II/III: Radical excision, laser treatment, biologic agents

Topical antimicrobial creams and grafts, flaps and secondary intention healing are advised.

  • Prognosis:
    High recurrence rates up to 50%
119
Q

Management of Toxic Epidermal Necrolysis (TEN)?

A

TEN

  • Autoimmune reaction to stimuli resulting in structural defects at the epidermal-dermal junction
  • Follow a prodromal period similar to URTI
  • Symmetrical macular eruption: face–> trunk–> extremities
  • Nikolsky sign: Lateral pressure causes epidermis to detach from basal layer.
  • Blisters form as extensive SPT injury with exposed dermis
  • Tx: Withdraw drug, supportive care, IVIG
120
Q

How to differentiate TEN from SJS?

A

TBSA Involved:
<10%: SJS
>30%: TEN
10-30%: SJS-TENS

121
Q

Management of Extramammary Paget’s Disease?

A
  • Rare adenocarcinoma of the apocrine glands (perianal, axillary, genitalia)
  • Eczema-like appearance
  • High incidence of concomittant other malignancies.
  • Tx: Resection to negative margins and adjuvant RT.
122
Q

Staging of wounds/pressure ulcers?

A

Stage I: Nonblanching erythema over intact skin

Stage II: Partial-thickness injury (epidermis/dermis) with blister/crater

Stage III: Full-thickness injury extending down to but not including fascia and without undermining adjacent tissue

Stage IV: Full-thickness skin injury with destruction/necrosis of muscle, bone, tendon, or joint capsule.

Tissue pressures that exceed pressure of the microcirculation (30mmHg) cause tissue ischemia.

Muscle is more suspectible to ischemia than skin (increased metabolic demand):

Areas at Risk:
Ischial tuberosity 28%
Trochanter 19%
Sacrum 17%
Heel 9%
123
Q

Management of Necrotizing Soft Tissue Infections?

A
TYPES
I: 80% are polymicrobial
- Most common pathogens:
--Bacteroides fragilis
--Eschericia coli

II: Group A beta-hemolytic strep

III: Gram negative marine organisms (most common is VIBRIO VULNIFICUS)

Tx: Broad spectrum antibiotics and surgical debridement

LRINEC Score:

1) WBC count
2) Hgb
3) CRP
4) Glucose
5) Creatinine
6) Na+

124
Q

Differentiate the different types of cystic skin lesions.

A

EPIDERMAL INCLUSION CYST

  • Most common
  • Completely mature epidermis + granular layer
  • Creamy material: Keratin from desquamated cells

TRICHILEMMAL CYST

  • Scalp, no granular layer
  • Females

GANGLIONS
- Connective tissue from synovial membrane of a joint/tendon sheath
- 60% on dorsal wrist, region of scapholunate ligament
- Tx: For asymptomatic px, just ASPIRATE.
For failed aspiration, EXCISE.

DERMOID CYST

  • Along body fusion planes
  • Midline abdomen/sacrum, occiput, nose, bone/tooth/nerve tissue.
  • Congenital, with squamous epithelium, eccrine glands, pilosebaceous units

PILONIDAL CUST

  • Acquired
  • From embedded hairs in the intergluteal cleft
  • MALES > F (2-4x risk)
125
Q

Diagnosis of Neurofibromatosis?

A

NF1 is Von Recklinghausen Disease.

  • Autosomal dominant
  • NF1 gene on chromosome 17q11.2 encodes neurofibromin protein (neuroectodermal differentiation, cardiac development)
DIAGNOSIS 
≥2 OF 8:
1) Cafe au lait spots ≥6 
> 5mm (for <10yo)
>15mm (for adults)

2) Neurofibromas (≥2) - MOST COMMON (3-15% lifetime risk for malignancy)

3) Axillary/inguinal freckling
4) Lisch nodules (≥2 iris hamartomas)
5) Optic nerve gliomas
6) Sphenoid dysplastic/long bone abnormalities
7) Cutaneous, subcutaneous, visceral NF
8) 1st deg relative with NF

  • PET Scan differentiates benign from malignant tumors.
126
Q

What type of collagen makes up dermal fibers?

A

Type I and III collagen

127
Q

A 23-year-old female is referred to you from her primary care physician’s office with the diagnosis of malignant melanoma with a 0.3 mm thickness diagnosed by a shave biopsy performed in the office. What is your treatment plan?

a. Excision with 5 mm margin
b. Excision with 20 mm margin
c. Excision with 30 mm margin and sentinel lymph node biopsy
d. Excisional biopsy

A

d. Excisional biopsy

128
Q

A 34-year-old Caucasian male presents with an exophytic lesion in an old burn scar on his right shin. What is the MOST appropriate treatment for this patient?

a. radiation therapy
b. Mohs’ surgery
c. wide excision
d. cryotherapy

A

c. wide excision

129
Q

A patient has a 5-year history of an enlarging mass at the right posterior forearm. On physical examination the mass was found to be 5 cm in diameter, hard and fixed. There was also note of wrist drop. What diagnostic imaging modality would you ask for?

a. x-ray
b. CT scan
c. MRI
d. ultrasound

A

c. MRI

130
Q

A patient with an 8 cm hard, fixed mass located at the right forearm was noted to have lost motor strength in the right hand. What would you recommend as the primary treatment for this patient?

a. Stage III basal cell cancer
b. Stage II malignant melanoma
c. Stage II squamous cell cancer
d. Stage III liposarcoma

A

b. Stage II malignant melanoma

131
Q

A 60-year-old woman comes to her physician because she has had lower back pain for 1 month. On PE, there are no remarkable findings except for pain on deep palpation of the abdomen. Findings from a routine urinalysis, CBC, and serum electrolyte panel all are unremarkable. Twenty years earlier, she was treated for Hodgkin’s lymphoma with abdominal irradiation and chemotherapy; there has been no evidence of recurrence during regular follow-up visits. MRI now shows a 10-15cm ovoid mass of the left retroperitoneum. Which of the following is most likely to be found in the patient’s retroperitoneum?

A. Desmoid tumor

B. Recurrent Hodgkin lymphoma

C. Rhabdomyosarcoma

D. Leiomyosarcoma

E. Malignant fibrous histiocytoma

A

E. Malignant fibrous histiocytoma

132
Q

A 6-year-old boy complains of discomfort in the right upper neck that has worsened over the past 6 months. On physical examination, a 5cm, firm mass is palpable in the right lateral neck. The mass is not painful or warm. The histologic appearance of this mass is shown in the figure. Which of the following immunohistochemical stains is most likely to be positive in the cells of this lesion?

A. Vimentin

B. Neuron-specific enolase

C. Cytokeratin

D. Factor VIII

E. CD3

A

A. Vimentin

Vimentin: intermediate cytoplasmic filament, immunoperoxidase staining sarcomas mark with antibody to vimentin.

NSE: with neural differentiation

Cytokeratin: epithelial origin (eg carcinomas)

Factor VIII: Endothelial related

CD3: T lymphocytic marker

133
Q

A 47-year-old man sees the physician because he has had dull constant pain in the midsection of the right thigh for the past 4 months. On PE, there is pain on palpation of the anterior right thigh, which worsens slightly with movement. The right thigh appears to have a larger circumference than the left thigh. A radiograph of the right upper leg and pelvis shows no fracture, but there is an ill-defined soft tissue mass anterior to the femur. MRI shows a 10 x 8 x 7cm solid mass deep to the quadriceps, but it does not involve the femur. What is the most likely diagnosis?

A. Nodular fasciitis

B. Liposarcoma

C. Osteosarcoma

D. Rhabdomyosarcoma

E. Hemangioma

F. Chondrosarcoma

A

B. Liposarcoma

NODULAR FASCIITIS
reactive fibroblastic lesion of young adults, usually on the upper extremities and trunk, can develop several weeks after trauma

LIPOSARCOMA
located in deep soft tissues and can be indolent; large size; most common sarcomas of adulthood

OSTEOSARCOMA
Younger than 20 years old; typically arise in the metaphysis

RHABDOMYOSARCOMA
Children, most often a tumor of head and neck, genitourinary, or retroperitoneum

HEMANGIOMA
In extremities, they tend to be small circumscribed skin lesions

CHONDROSARCOMA
Seen over a wide age range, but arise within bone

134
Q

A “sebaceous cyst” is removed from the scalp of a 48-year-old woman. Which of the following would be expected on histologic exam?

A. Presence of sebum

B. Presence of a granular layer

C. Presence of eccrine glands

D. Presence of epidermis covered by an external basal layer.

A

D. Presence of epidermis covered by an external basal layer.

EPIDERMAL
Most common type of cutaneous cyst, and may present as a single, firm nodule anywhere on the body; have a mature epidermis complete with a granular layer
Substance is actually KERATIN
Cyst walls consist of an epidermal layer oriented with the basal layer superficial, and the more mature layers deep (ie, with epidermis growing into the center of the cyst)

The desquamated cells (keratin) collect in the center to form the cyst.

DERMOID
Congenital lesions that result when epithelium is trapped during fetal midline closure
Eyebrow is most frequent site of presentation, but dermoid cysts are common anywhere from nasal tip to forehead
Demonstrate squamous epithelium, eccrine glands, ilosebaceous units
May develop bone, tooth, or nerve tissue on occasion.

TRICHELEMMAL
AKA Pilar cysts, the second most common cutaneous cyst, occur more often on the scalp of females
Cyst walls do NOT contain a granular layer
Contain a distinctive outer layer resembling the root sheath of a hair follicle (trichilemmoma)

135
Q

Which of the following is the most common form of basal cell carcinoma?

A. Morpheaform

B. Superficial spreading

C. Pigmented

D. Nodular

A

D. Nodular

136
Q

Which type of melanoma has the best overall prognosis?

A. Superficial spreading

B. Nodular

C. Lentigo maligna

D. Acral lentiginous

A

C. Lentigo maligna

137
Q

A patient presents with a biopsy-proven melanoma of the thigh which is 3mm thick on histologic examination. At the time of excision, how wide should the margins be?

A. 1cm

B. 2cm

C. 3cm

D. 4cm

A

B. 2cm

138
Q

Which of the following is appropriate treatment following wide local excision of a 4cm leiomyoma of the calf with negative margins and no metastases?

A. Radiation therapy

B. Systemic chemotherapy

C. Isolated limb perfusion of chemotherapy

D. Radiation therapy and chemotherapy

A

A. Radiation therapy

Stage I (Low-grade tumors): Surgical resection

Stage II (≤5cm, high-grade tumors): Surgery +/- RT

Stage III (>5cm, high-grade or nodal disease): Surgery, RT +/- Chemotherapy

Stage IV (Distant metastatic disease): Chemotherapy +/- Surgery

Small (<5cm) primary tumors with no evidence of distant mets are managed with local therapy consisting of surgery, alone or in combination with RT, when wide pathologic margins are limited because of anatomic constraints.

139
Q

Which of the following sarcomas is most responsive to chemotherapy?

A. Leiomyosarcoma

B. Chondrosarcoma

C. Liposarcoma

D. Synovial sarcoma

A

D. Synovial sarcoma

HIGH SENSITIVITY TO CHEMO:
Synovial sarcoma
Myxoid/round cell

INTERMEDIATE SENSITIVITY TO CHEMO:
Liposarcoma
Myxofibrosarcoma
MPNST
Angiosarcoma
Desmoplastic 
Round cell

POOR SENSITIVITY (RESISTANT) TO CHEMO:
GIST
Chondrosarcoma

140
Q

The most appropriate surgical treatment for a 2cm leiomyosarcoma of the greater curvature of the stomach is

A. Local resection with 3cm margin of normal tissue

B. Subtotal gastrectomy

C. Total gastrectomy

D. Total gastrectomy and lymphadenectomy

A

A. Local resection with 3cm margin of normal tissue

Lymphatic spread is NOT the primary route of mets for GI sarcomas.

Lymphadenectomy is not routinely performed as part of resection.

General recommendation is to perform a margin-negative resection with 2-4cm margins of normal tissue.

141
Q

A 75-year-old farmer complained of a scaly, plaque like skin lesion on his forearm with recent development of ulceration. Biopsy reveals invasive squamous carcinomas within actinic keratosis negative for examination of axillary nodes. Definitive treatment is:

(A) Local wound care until the ulcer heals; then wide excision and repair

(B) Excision of the lesion with frozen section determined free margins and repair

(C) Wide excision; split-thickness skin graft and axillary node dissection

(D) Wide excision; split-thickness graft and radiation therapy

(E) Wide excision; split-thickness graft and chemotherapy

A

B

Actinic (solar) keratosis is the most common premalignant lesion usually seen in older, light- complexioned individuals.

The incidence of degeneration to invasive squamous carcinoma is 20–25%.

These carcinomas, arising from actinic keratosis, metastasize suggesting conservative excision in treating them.

142
Q

A 65-year-old light-complexioned male presents with a solitary scaly plaque like lesion on his forearm present for many years. The lesion is 0.5 cm in diameter. Shave biopsy reveals intraepithelial squamous cell carcinoma (Bowen’s disease), incompletely excised. Further treatment includes:

(A) Wide excision of the lesions and sentinel node biopsy

(B) Referral for local radiation therapy

(C) Excision and repair of this area, ensuring clear surgical margins

(D) No further treatment indicated

(E) Local application of 5-fluorouracil (5-FU) cream

A

C

Bowen’s disease represents an intraepithelial squamous cell carcinoma (carcinoma in situ) and is seen in older patients. These lesions tend to have a long clinical course.

Adequate excision is the recommended treatment as these lesions can become invasive squamous cell carcinomas and metastatasize.

143
Q

A 45-year-old soccer player presents with a 6-month history of an ulcerative nodular lesion, 1.5 cm in diameter in the region of the right oral commissure. Biopsy reveals basal cell carcinoma. The preferred treatment is:

(A) Mohs micrographic surgery and
subsequent reconstruction

(B) Excision with a clinical margin and local
flap repair

(C) Topical 5-FU

(D) Local radiation therapy

(E) Cryotherapy

A

(A) Basal cell carcinoma is the most common malignancy in caucasians. The lesion is cured by complete excision and reconstruction (Moh’s) surgery.

144
Q

A 43-year-old window cleaner fell off a scaffold. He sustained an open wound on the right leg. Debridement was carried out in the emergency department, and the edges of the wound were left open. The wound measures 4 cm × 6 cm. What is TRUE of wound contraction?

(A) It occurs within 12 hours of injury.

(B) It is more prominent over the tibia than gluteal region.

(C) It is accelerated if wound is excised 3 days after injury.

(D) It accounts for excessive fibrous tissue formation and fixation of tissue around a joint.

(E) It is experimentally less affected by excision of tissue from center of wound rather than at the periphery

A

E

Wound contraction refers to the decrease in diameter of an open wound. It commences on about the fourth day after injury and continues at a relatively rapid rate (1/2–1 mm/d). It is maximal in areas where tissue laxity exists.

Wound contraction should not be confused with wound contracture where scar formation over a joint interferes with mobility.

Experimentally, it less affected by excision of tissue from the center of the wound, rather than at the periphery.

145
Q

A 43-year-old window cleaner fell off a scaffold. He sustained an open wound on the right leg. Debridement was carried out in the emergency department, and the edges of the wound were left open. The wound measures 4 cm × 6 cm.

Which factor is least likely to inhibit wound contraction?

(A) Radiation

(B) Cytolytic drug

(C) Transformation growth factor b

(D) Full-thickness skin graft

(E) External splints

A

D

Following the application of a full-thickness graft, contraction at the site of the recipient site is maximally inhibited by a full-thickness and to a lesser extent by the partial-thickness graft.

146
Q

A 43-year-old male undergoes a total proctocolectomy for ulcerative colitis. The terminal ileum is brought out on the anterior abdominal wall as an end (Brooke) ileostomy. What is necessary to obtain optimal healing?

(A) The ileostomy should be circular rather than square.

(B) The seromuscular layer is sutured to the epithelium of the skin to avoid inflammatory changes.

(C) The ileostomy must be constructed to avoid fixing the mesentery.

(D) The mesentery of the ileal loop should be widely cut to increase its mobility.

(E) The ileostomy must be constructed on the right side.

A

A

The ileostomy should be circular rather than square to avoid excessive stenosis of the stoma. Wound healing by a square incision results in a greater degree of stenosis than by an equivalent circular stoma.

Failure to close the gap between the ileal loop on the abdominal wall may lead to subsequent internal herniation.

It is critical to ensure that the ileal stump is not devascularized.

147
Q

A 64-year-old male is to undergo an elective laparotomy procedure. The proposed wound is considered as “clean-contaminated.”

This term implies an infection rate of which of the following?
(A) 1%
(B) 2%
(C) 9%
(D) 15%
(E) 30%
A

C

In a clean wound, the anticipated infection rate should be 1.5–5%, in a contaminated wound, 15%, and a dirty wound, 30–40%.

148
Q

A 64–year–old male is to undergo an elective laparotomy procedure. The proposed wound is considered as “clean–contaminated.” The wound characteristic indicates which of the following?

(A) Entry of intestinal or urinary tract without significant spillage

(B) Gross spillage from intestinal tract

(C) No entry of intestinal tract

(D) Entry into infected tissue

(E) Drainage of an abscess

A

A

If spillage is substantial or infected tissue has entered, the wound is classified as contaminated.

Dirty wounds are used for drainage of an abscess or debridement of infected tissue.

149
Q

A 56-year-old male is burned while sleeping in his home. His right upper and lower extremity and the anterior aspect of the upper chest have extensive second-degree burns.

A second-degree burn is characterized by which of the following?

(A) Coagulative necrosis extending to
subcutaneous fat

(B) Pearly white appearance

(C) Anaesthetic

(D) Erythema and bullae formation

(E) Requires immediate skin grafting

A

D

In a second-degree burn, the skin appendages in the dermis are minimally destroyed (superficial partial thickness) or more extensively destroyed (deep partial thickness).

In a third-degree (full-thickness) burn, all of the dermis, with skin appendages, are destroyed, and the lesion extends to the subcutaneous fat layer.

150
Q

A 56-year-old male is burned while sleeping in his home. His right upper and lower extremity and the anterior aspect of the upper chest have extensive second-degree burns.

The extent of the burn is calculated to represent what percentage of body surface area (Fig. 2–1)?

(A) 10%

(B) 20%

(C) 30%

(D) 40%

(E) 50%

A

D

In calculating burn surface area, the rule of “9’s” assigns 9% to each upper extremity, 18% to each lower extremity, and 9% to the head and neck. The trunk and abdomen (36%) is divided into four equal parts (9% each). Thus, upper trunk anteriorly would be 9%.

151
Q

Following initial resuscitation, based upon the Parkland formula, the patient was resuscitated with Ringer’s lactate solution at 800 mL/h. Further assessment after 6 hours reveals oliguria. What should the next step in management be?

(A) Continue with increased amount of lactated Ringer’s solution

(B) Give Plasma

(C) Give Diuretics to improve urine flow

(D) Colloid solution

(E) Continue initial resuscitation with normal saline

A

A

Continue with increased amount of lactated Ringer solution. Urine flow should be 0.5–1.0 mL/kg/h.

Patients exposed to inhalation on burns, and those admitted following alcoholic intoxication require additional fluids.

In general, for second- and third-degree burns, the Parkland formula is used to administer 4 mL/kg weight of patient × percentage of area of burn.

Half of the calculated amount is given within 8 hours and the remainder during the subsequent 16 hours.

152
Q

After a period of resuscitation, management of this patient should include which of the following?

(A) Tangential excision of all eschar until bleeding is encountered

(B) Split-thickness graft (Fig. 2–2) if wound grows b-hemolytic streptococci

(C) Use of cadaver allograft when required

(D) Avoid use of porcine xenograft

(E) Chest x-ray useful for diagnosis of inhalation injury

A

C

Use cadaver allograft when required.

Tangential excision of the skin (to secure a bleeding surface) is done with a guarded dermatome. However, because of possible extensive blood loss, it should be limited to an area <20% of the total body surface area.

The presence of bacteria growth >105 organisms/cm2 or growth of beta-hemolytic streptococci should contraindicate split-skin-thickness grafting.

153
Q

A 12-year-old boy has multiple skin lesions that are diagnosed as von Recklinghausen’s syndrome (NF-1). What is TRUE of this condition?

(A) It does not show other malignant lesions.

(B) It is autosomal recessive.

(C) It is associated with optic nerve gliomas.

(D) It is characterized by atrioventricular
(AV) malformation.

(E) It is associated with dermoid

A

C

It is inherited as a autosomal dominant disorder and noted in nearly 1/5000 births.

The NF-1 gene encodes a protein neurofibromin that plays a role in neuroectodermal differentiation and cardiac development.

154
Q

A 29-year-old female swimmer develops a pigmented lesion on the right thigh. With reference to a pigmented lesion, there is an increased risk of developing melanoma if it is identified with which of the following?

(A) Hutchinson freckle (lentigo maligna)

(B) Freckle involving basal layer of skin

(C) Congenital nevocellular nevi

(D) Hemangioma

(E) Tophi

A

C

Most melanoma arise from nondysplastic nevi. Congenital nevocellular nevi found in about 1/100 births have a 3–5% lifetime risk of undergoing malignant change.

Dysplastic (atypical) nevi may be familial and predisposed to malignancy.

Hutchinson freckle occurs mainly in older patients.

155
Q

A 67-year-old business executive and tennis player has a basal cell carcinoma removed from the right cheek. What is TRUE of basal cell carcinoma (Fig. 2–3)?

(A) It may show a flat ulcer.

(B) It may metastasize to lymph nodes.

(C) It may metastasize to remote skin areas.

(D) It is found exclusively in the head and neck.

(E) It is best treated by topical 5-FU.

A

A

The surface of a basal cell carcinoma has a shiny appearance with telangiectasia. Ulcer formation may occur; hence, are named rodent ulcer.

Although treatments with 5 FU, cryosurgery, or electrodessication are effective in treatment, surgical excision offers the best results and ensures an accurate diagnosis.

156
Q

A 38-year-old female undergoes removal of a 2×1cm skin lesion shown to be a melanoma. It is reported as Clark level 1, which implies what?

(A) It is superficial to the basement membrane.

(B) It is 1 mm in thickness.

(C) It has nodal involvement.

(D) It involves the papillary layer.

(E) It involves the reticular dermis.

A

A

Level II  (≤0.75mm) involves the papillary dermis (T1);
Level III (0.76-1.5mm) between the papillary and reticular dermis (T2);
Level IV (1.51-4mm) the reticular dermis (T3); and 
Level V (≥4mm) the subcutaneous fat (T4). 

The Breslow classification utilizes differences in the thickness of the tumor.

157
Q

A 49-year-old male postman had undergone several operations to excise recurrent infections in both axillary lesions and perianal region. The lesions are hidradenitis suppurativa (Fig. 2–4).

Which is TRUE of these?

(A) They arise from stratum corneum of skin.

(B) They are noninflammatory conditions.

(C) They always require surgical intervention.

(D) They frequently involve the scalp.

(E) They are usually caused by staphylococci and streptococci.

A

E

Usually caused by staphylococci and streptococci. Hidradenitis suppurativa is an infection of the apocrine glands and surrounding subcutaneous tissue and fascia, which most commonly involves the axilla, groin, perineum, and perianal region. The periumbilical and areola region may be involved.

In milder cases, local hygienic measures and tetracycline may be adequate; in more severe cases, wide excision is indicated.

158
Q

List the layers of skin from the most superficial to the deepest layer adjacent to the dermis:
(a) basal layer, (b) granular layer, (c) prickle layer,
and (d) stratum corneum.

(A) a b c d
(B) d b a c
(C) d c b a
(D) c a b d
(E) c a d b
A

C

The stratum corneum consists mainly of dead cells and keratin.

159
Q

A 12-year-old boy has multiple skin lesions that are diagnosed as von Recklinghausen’s syndrome (NF 1). What is TRUE of this condition?

(A) It does not show other malignant lesions.

(B) It is autosomal recessive.

(C) It is associated with optic nerve gliomas.

(D) It is characterized by AV malformation.

(E) It is assocated with dermoid.

A

C

It is inherited as a autosomal dominant disorder and noted in nearly 1/5,000 births.

The NF-1 gene encodes a protein neurofibromin that plays a role in neuroectodermal differentiation and cardiac development.

160
Q

A 35-year-old White male previously diagnosed with basal cell nevus syndrome (Gorlin’s syndrome) presents with a new lesion for treatment. Apart from multiple basal cell lesions other features of this disorder may include the following, EXCEPT:

(A) The disorder is genetically determined and transmitted as an autosomal dominant.

(B) Rib abnormalities such as splayed or bifid ribs.

(C) Skin ribs on the palms and soles.

(D) A benign clinical course before puberty.

(E) Normal mental development.

A

E

Gorlin’s syndrome is genetically determined, a disorder of childhood onset. Along with multiple basal cell carcinomas, other abnormalities include skin ribs on the palms and soles, epithelial jaw line cysts, rib abnormalities, ectopic calcifictions in the dura, and mental retardation.

The disease is transmitted as autosomal dominant with no sex linkage.

Generally the tumors have a benign clinical course until after puberty.

161
Q

An elevated lactate dehydrogenase level in a 47-year-old male with abdominal mass, fever and night sweats, suggests a

A. Germ cell tumor

B. Lymphoma

C. Liposarcoma

D. GIST

A

B. Lymphoma

162
Q

Which statement is true regarding retroperitoneal sarcomas?

A. Retroperitoneal leiomyosarcomas usually metastasize to the liver.

B. Tumor grade does NOT equate with positive outcome for patients with retroperitoneal sarcoma.

C. Complete surgical resection was achieved in almost 80% of the cases

D. Retroperitoneal sarcomas recur in 1/3 of patients.

A

A. Retroperitoneal leiomyosarcomas usually metastasize to the liver.

163
Q

Which statement is true regarding retroperitoneal sarcomas?

A. Most retroperitoneal tumors are benign

B. Locoregional recurrence is uncommon

C. They generally present as large masses usually >20cm at time of diagnosis

D. Evaluation of a patient should include accurate history involving presence of fever and night sweats

A

C. They generally present as large masses usually >20cm at time of diagnosis